OK…I want numbers. What is the probability the universe is the result of chance?

Posted on December 1, 2010 By

“A common sense interpretation of the facts suggests that a superintellect has monkeyed with physics, as well as with chemistry and biology, and that there are no blind forces worth speaking about in nature. The numbers one calculates from the facts seem to me so overwhelming as to put this conclusion almost beyond question.”

–Cambridge University astrophysicist and mathematician Fred Hoyle

.

“Fred Hoyle and I differ on lots of questions, but on this we agree:  a common sense and satisfying interpretation of our world suggests the designing hand of a superintelligence.”

–Former Harvard University Research Professor of Astronomy and the History of Science Owen Gingerich, who is now the senior astronomer at the Smithsonian Astrophysical Observatory. Gingerich is here reflecting on Fred Hoyle’s above comment.

 —————————————-

 

The reader of the essay entitled Is There A God (What is the Chance the World is the Result of Chance?) may be interested in knowing some hard numbers with regard to the probability that the universe occurred randomly (i.e. no conscious creator involved). Oxford University Professor of Mathematics John Lennox quotes renowned Oxford University mathematical physicist Roger Penrose:

“Try to imagine phase space… of the entire universe. Each point in this phase space represents a different possible way that the universe might have started off. We are to picture the Creator, armed with a ‘pin’ — which is to be placed at some point in phase space… Each different positioning of the pin provides a different universe. Now the accuracy that is needed for the Creator’s aim depends on the entropy of the universe that is thereby created. It would be relatively ‘easy’ to produce a high entropy universe, since then there would be a large volume of the phase space available for the pin to hit. But in order to start off the universe in a state of low entropy — so that there will indeed be a second law of thermodynamics — the Creator must aim for a much tinier volume of the phase space. How tiny would this region be, in order that a universe closely resembling the one in which we actually live would be the result?”

Lennox goes on to cite Penrose’s answer:

“His calculations lead him to the remarkable conclusion that the ‘Creator’s aim’ must have been accurate to 1 part in 10 to the power of 10 to the power or 123, that is 1 followed by 10 to the 123rd power zeros.”

As Penrose puts it, that is a “number which it would be impossible to write out in the usual decimal way, because even if you were able to put a zero on every particle in the universe, there would not even be enough particles to do the job.”

And the only alternative to the universe arising from chance is for it to have arisen deliberately. Deliberate action requires a conscious creator (read: God). And for those who are still tempted to conclude that our universe is just the result of a very extremely improbable accident, I explain in Why God? Why not just plain luck? why bare probability (chance), alone, can never cause anything….let alone the creation of a universe.


  1. I really find it amazing when its put in number form. I have struggled my whole life about were my heart really is. I decided to face my fears of being proven wrong and investigate it for my self. What i truly love about this site is that alot of the material that points to God is discovered by atheists, agnostics, and beleivers. I honestly beleive that information such as this should be taught in church. Im not talking about the main sermon but in Sunday school. I thank you Scott for letting the Lord work through you to create this site.

  2. Dan Silva says:

    I have found that the books and information provided in this website to be powerful tools when defending the truth about God. My debates now start from a science point of view rather than a faith base perspective. Those who I wish to persuade to the reality of God are asking much more questions as a result of these new findings for my belief. I just finished There is a God… How the world’s Most notorious atheist changed his mind…by Antony Flew….I read it in two day and am now feasting on The Hidden Face of God… Science Reveals the Ultimate Truth…. by MIT physicist and biblical scholar Gerald Schroeder…..I’m so glad to have found this site…

    • Dan:

      That is how it started for me: Once I began to learn how much good information there is on this subject matter that does not reach mass circulation, I developed a voracious appetite for learning more.

      And the more I learned, the more it became apparent that there is an enormous need to get this information into the hands of more people. You can bet your bottom dollar that the media and academia will not disseminate this information. And that is the beauty of the internet…it allows us to go under the fence that is set up by the highly secularized media and academia.

      Scott

    • Darren says:

      Can I be directed to where these numbers come from? I am looking for how this probability was determined. Preferably a source that demonstrates how these numbers were determined.

      • Scott Youngren says:

        Darren,

        If you want to know where the numbers come from, I recommend that you read Penrose’s book titled The Emperor’s New Mind.

        Scott

    • Jay says:

      The problem with the entire concept is that you have to assume that we are the pinnacle of creation. That one pin point in any other direction could not have resulted in us developing in any better design. The odds of the universe ending up completely void of life would be the exact same odds of it ending up the way it is, in the same way that dealing out all the same suit from a shuffled full deck of cards is the same as dealing out any combination of cards in one try.

      • Scott Youngren says:

        Jay,

        No, there is no need to assume that we are the pinnacle of creation. Life in any form would not exist if several “anthropic constants” were not very very precisely balanced. There are roughly 122 such anthropic constants. A copy and paste from this article, which lists just 15 of them:

        Anthropic Constant 1: Oxygen Level
        On earth, oxygen comprises 21 percent of the atmosphere. That precise figure is an Anthropic Constant that makes life on earth possible. If oxygen were 25%, fires would erupt spontaneously, if it were 15%, human beings would suffocate.

        Anthropic Constant 2: Atmospheric Transparency
        If the atmosphere were less transparent, not enough solar radiation would reach the earth’s surface. If it were more transparent we would be bombarded with far roo much solar radiation down here. (In addition to atmospheric transparency, the atmospheric composition of precise levels of nitrogen, oxygen, carbon dioxide and ozone are in themselves Anthropic constants).

        Anthropic Constant 3: Moon-Earth Gravitational Interaction
        If the interaction were greater than it currently is, tidal effects on the oceans, atmosphere, and rotational period would be too severe. If it were less, orbital changes would cause climatic instabilities. In either event, life on earth would be impossible.

        Anthropic Constant 4: Carbon Dioxide level
        If the CO2 level were higher than it is now, a runaway greenhouse effect would develop (we’d all burn up). If the level were lower than it is now, plants would not be able to maintain efficient photosynthesis (we’d all suffocate).

        Anthropic Constant 5: Gravity
        If the gravitational force were altered by 0.00000000000000000000000000000000000001 percent, our sun would not exist, and, therefore neither would we. Talk about precision.

        Anthropic Constant 6: Centrifugal Force
        If the centrifugal force of planetary movements did not precisely balance the gravitational forces, nothing could be held in orbit around the sun.

        Anthropic Constant 7: Rate Of Expansion
        If the universe had expanded at a rate one millionth more slowly than it did, expansion would have stopped and the universe would have collapsed on itself before any stars had formed. If it had expanded faster, then no galaxies would have formed.

        Anthropic Constant 8: Speed Of Light
        Any of the laws of physics can be described as a function of the velocity of light (now defined to be 299,792,458 meters per second). Even a slight variation in the speed of light would alter the other constants and preclude the possibility of life on earth.

        Anthropic Constant 9: Water Vapor Levels.If water vapor levels in the atmosphere were greater than they are now, a runaway greenhouse effect would cause temperatures to rise too high for human life. If they were less, an insufficient greenhouse effect would make the earth to cold to support human life.

        Anthropic Constant 10: Jupiter.
        If Jupiter were not in it’s current orbit, the earth would be bombarded with space material. Jupiter’s gravitational field acts as a cosmic vacuum cleaner, attracting asteroids and comets that might otherwise strike earth.

        Anthropic Constant 11: The Earth’s Crust.
        If the thickness of the earth’s crust were greater, too much oxygen would be transferred to the crust to support life. If it were thinner, volcanic and tectonic activity would make life impossible.

        Anthropic Constant 12: The Earth’s Rotation.
        If the rotation of the earth took longer than 24 hours, temperature differences would be too great between night and day. If the rotation period were shorter, atmospheric wind velocities would be to great.

        Anthropic Constant 13: Axis Tilt.
        The 23-degree axis tilt of the earth is just right. If the tilt were altered slightly, surface temperatures would be too extreme on earth.

        Anthropic Constant 14: Atmospheric Discharge.
        If the atmospheric discharge (lightning) rate were greater, there would be too much fire destruction; if it were less there would be little nitrogen fixings in the soil.

        Anthropic Constant 15: Seismic Activity.
        If there were more seismic activity, much more life would be lost; if there were less, nutrients on the ocean floors and in river runoff would not be cycled back to the continents through tectonic uplift. (yes, even earthquakes are necessary to sustain life as we know it).

      • Tony says:

        Not true. The reason the odds of dealing any suit in a deck of cards is equal across the board is because the cards already exist to be chosen from. By purely naturalistic science (as these odds are pertaining to) the universe was preceeded by nothing (no space, time, or matter; nothing “physical”). If there was nothing, then there were no “cards” (possibilities) to be chosen from, or chanced.

  3. David Platek says:

    Gentlemen,

    Sorry to burst all of your ‘bubbles of comfort’, but you have all fallen into (or failed to get out-of) the same trap of circuitous reasoning and boot strapped arguments that you mistake for ‘logic’, all of which continues to take a vast majority of otherwise intelligent and rational humans round-and-round without any real ‘advance’ towards greater enlightenment on a far more important subject, that being, the nature of intelligence, consciousness, and the universe itself.
    I do not profess to have an ‘answer’ as to whether there is a god or a creator, and I think any debate at this stage of our collective ignorance wastes otherwise useful intellectual effort that might actually move us closer towards a more probable understanding upon which we can all agree, i.e.: the then currently accepted ‘truth’. The discussion has been and seems to remain just as pointless as when it was first offered for determination as to how may angels can dance on the head of a pin. Although I am not aware of any government grant or ‘faith based’ private investor willing to fund the project, I would be willing to bet my donuts against their dollars that even if all the currently available pins throughout the world could be simultaneously examined by a legion of calibrated and certified instruments employing Non Contact Atomic Force Microscopy (AFM), a new IBM technique that allows imaging down to the resolution of the individual energy bonds that bind atomic structures (See: http://www-03.ibm.com/press/us/en/pressrelease/38856.wss ), not one, not even ONE! ‘angel’ would be found dancing (or doing anything else) on any of the atoms of any of the pins.
    I would also be willing to make a corollary wager that, as I was counting my winnings from the first, the same ‘faith-based-suckers’ whom I had just taken to the cleaners would signify their ‘faith’ by proclaiming that the results of the science ‘didn’t prove that angels don’s exist, only that they must be ‘invisible’ ” or some other similar nonsense. These sorts of responses from the faith based communities, ALL OF THEM, are reflective of a denial that runs so deeply in their minds that most of them are willing to die and send their children to die in defense of the mumbo-jumbo that propagates from their ‘teachings’ and ‘holy books’.
    In performing root cause analysis in the search for understanding as to why this curiously irrational behavior seems to permeate the species, showing little to no signs of subsiding (or evolving) into a more calm, rational and logical thought process, I have come to conclude that consciousness and ‘self-awareness’ are each their own nemesis. We construct ‘higher power’ or ‘afterlife’ scenarios in order to give purpose and meaning to our lives and protect us from having to accept the realization that these notions may very well be purely fanciful and that: heaven, hell, God, Ala, reincarnation, etc., simply DO NOT EXIST. We seem to have a fatalistic psychological ‘need’ to believe in Santa Claus type things and continually re-assure ourselves that they DO, in fact ‘exist’, in order to try to avoid or at least suppress the chaos (used here in its classical sense) havoc, mayhem and barbaric behavior we would undoubtedly wield upon each other should the ‘truth’ be otherwise (hmmmm, just as a thought, how’s that ‘do-it-for-the-after-life’ stuff doing so far? Seems to me that just a quick look around the world proves my point. Seems to me that all of you: Christians, Jews, Muslims and all the rest, must have at least SOME PART of it wrong…). You see, it is the validity of the major premise itself that lies at the heart of the debate. The clamoring over the existence or non-existence of ‘god’, just like the debate about the angles, completely misses the point. No progress will be made in furthering our journey towards a higher understanding of our world, our universe, or anything else, unless and until we all get comfortable with the idea that maybe, just maybe, we have no purpose on Earth except that which WE, as sentient beings possessing unbridled free will, define our purpose for ourselves (see, no ‘god’). And, that when we pass from life into death, the ONLY THING that will happen is that we will cease to exist.
    Throughout the known history of the human species on the planet Earth, solutions to problems and explanations for phenomena, whether directly observable or by inference only from a result, borne of the sound application of scientific principles employing a systematic, scientific approach have ALWAYS replaced the mumbo-jumbo of ‘faith based’ responses which had existed before them, ALWAYS, 100% of the time. To put it another way, in ALL of the known history of man, when called upon to do so, the published results of a scientific investigation into an inquiry where religion had previously proclaimed the answer has ALWAYS PROVED that the religious response was WRONG!
    Does anyone in the faith-based-community still want to tell anybody that the Earth is the center of the Universe? How about that the Sun and the stars revolve around the Earth? Or how about ‘we need to sacrifice a virgin to Chaac, the Mayan Rain God to bring rain’? Even the most affected in their respective communities would have to agree that if they chose to argue in favor of any of these propositions, the most probable result would be that they would get a nice long rest in a hospital with softly padded walls and soothing music. And so on and so on and so on, throughout every culture and religion on the planet since the dawn of time.
    The reality is this: religious ‘solutions’ to problems and any other ‘answer’ to a question that is based upon a ‘belief’ or ‘faith’ in a god or a higher power is nothing more than a placeholder for ignorance. That is, it is a ‘thing’ to plug into the void so that we can get on with our lives. It is a useful artifice that serves a laudable purpose. It is not, however, by any stretch of the imagination, an acceptable ‘answer’ to any question on any one of life’s seemingly endless series of examinations, although, until replaced by a well reasoned scientific alternative, it remains a psychologically necessary one for most. Acceptance of ignorance as an ‘answer’ to anything axiomatically and instantaneously leads to stagnation and puts the brakes on learning. It stops all progress and impedes further development. It leaves a person open and susceptible to harm as a victim of his or own self-imposed ignorance and creates opportunities for snake oil salesmen to continue to peddle their wares. Bowing to the East (or whatever hell direction they bow) at a specific time of day; kneeling, sitting, standing, then knelling then sitting while reading unfathomable phrases in unison; Looking towards the heavens while making the sign of a cross; pointing to the sky after a touchdown; taking one of the now copyrighted ‘Tebow Knee’ poses (I wonder if he is going to bring a class action lawsuit against the Catholic Church to collect royalties – I’m sure he will if it is a part of Jesus’ ‘plan’ for him); pushing the button on the bomb wrapped around your chest ‘for Ala’, and; all the rest of the mindless behavior that continues to this day, is, in my humble opinion, NO DIFFERENT from the ancient ritualistic behavior of religions, cults, and great supposed ‘civilizations’ long past at whose practices we so arrogantly scoff. In fact, I think that the implications for the continuation of such strikingly similar behavior is that, although participants in their respective ‘communities of ignorance’ will disagree, we, as a species, sure don’t seem to have learned very much.
    So, here is my advice, invitation and/or challenge: try taking a step outside of your comfort zone for a little while. Try, really try, to accept as a fact, not for real, but just for the sake of role-play and intellectual masturbation, that you DON’T MATTER. Try to imagine that the reality IS that:
    1. There is no ‘god’ or ‘creator’, at least in the form that you currently ‘believe’;
    2. the universe in which we live came into existence as a consequence of the interaction of googolplex upon infinite googolplex of strands of pure energy (‘strings if you like’) that obeyed the laws of quantum mechanics, existing in all possible energy states simultaneously; being in all possible dimensions simultaneously; exhibiting maximal entropy, completely devoid of any form, wholly ‘unobserved’, free from decoherence but ‘boiling’, ‘bubbling’ and ‘alive’ with the constant emergence of a particle and an arrow of time followed by the disappearance of both within one plank length, in any one of any and all of the dimensions (the quantum foam);
    3. Time does not pass, because time never comes into existence beyond the shortest measurable unit of time, dissolving nearly instantaneously back into the foam;
    4. The interactions, the formation of the particles, the arrows of time, the dimension they choose, are all governed by nothing more than the probability of their occurrence;
    5. The foam continues in this manner for an immeasurable period of time because there simply is no such thing ‘known’ to the quantum foam;
    6. Particles that form must ‘decay’ within a Plank length unit of time, because there is nothing in the foam to ‘observe them’ and cause them to maintain their transitory period of sufficient de-coherence to permit them to ‘exist’;
    IMPORTANT NOTE: (The equations that describe the second law of thermodynamics and entropy predict that, at some point in time, a pool of water (high entropy) WILL assemble itself into ice (low entropy). The exercise is dismissed in the classroom as a ‘curiosity’ because the ‘answer’ to the equation is a time frame greater than the known existence of the Universe. But in the world of creating something from nothing, the response of the quantum foam would most likely be, “so what?”)
    7. Within the proscribed number of events, the foam yields maximal entropy to the rules that govern it and ALL, or a staggeringly large number of ‘strings’ all chose to take the same ‘state’ including their ‘choice’ of dimension (3);
    8. As in the googolplex to the googolplex power of times that particles had formed, an arrow of time emerged. However, this ‘time’ (pardon the pun), a sufficient number of simultaneous choices were made such that the resulting ‘particle’ was capable of ‘observing itself’ with sufficient ‘force’ (for lack of a better word) to cause sustainable de-coherence and be able it to continue its existence as a particle in a three dimensional ‘space’ that had, just a plank instant before, not been in existence itself;
    9. Now that a sustainable arrow of time has emerged, the concept of time became a reality and the notion of ‘then and now’ took on meaning;
    10. At this ‘time’ there were no particles as we currently understand them and most certainly, there were no electrons, neutrinos, photons, or any other of the other particles that currently define the Standard Model;
    11. Since there were no photons, there was no light; since there was no space but for a plank sized ‘chunk’ in which the sustained de-coherence ‘event’ had chosen to occupy, there were no rules to stop ‘events’ from happening ‘faster than the speed of light’ (Einstein wasn’t ‘wrong’, but just as General Relativity enhanced our understanding by exposing the limitations of Newtonian Physics, Quantum theories are putting relativity into a new and more ‘enlightened’ context);
    12. The newly formed ‘stuff’, energy which has now taken on the properties of ‘matter’, has also become a powerful ‘observer’. Almost instantaneously (probably as long as the period of inflation that is now believed to have followed the Big Bang), the entropy of the quantum foam was lost, due to de-coherence, and any remaining ‘strings’ of energy, all of which were pulled into the three dimensional ‘space’ created by the event, fell under the watchful eye of the ‘particle’ and were ‘forced’ to make a ‘choice’ as to what state they wished to take. Some chose to be ‘the same’ and became entangled. Some ‘chose’ other dimensions. Still others ‘chose’ to be in the third dimension but took on another energy state.
    13. All of this happened in a nearly immeasurably small period of time, and, in an immeasurable small ‘space’ – this is the ‘state’ of the singularity from which we came.
    14. With infinite temperature and infinite energy (well, nearly infinite, remember, the ‘event’ has taken on properties of matter and is no longer a pure energy ‘wave’) all trying to occupy the same space and achieve an infinitely small level of entropy, the second law of thermodynamics and the collapsing waive functions that now describe the arrow of time, result in what we call ‘The Big Bang’, and, that ever since that ‘time’ the universe has been engaged in the process of progressing along the collapsing wave front of time towards regaining the entropy lost at the instant of creation; a journey that will take googolplex upon googolplex of time to complete. All matter in the universe will eventually reduce itself into an ever expanding and evolving void of three dimensional space. Regions of quantum foam, born of the final stages of decay into coherent strings of pure energy, will begin to appear and ‘swallow-up’ space itself. The arrow of time will continue until all useful energy is consumed in the process and the second law reclaims its property and restores the quantum foam from which it first appeared. Waiting, once again, for an immeasurable amount of a thing by which to measure which no longer exists, for the whole process to begin, all over again, without any knowledge, understanding, memory nor concern for the Universe that had ‘been’ before (Lookie here: ashes to ashes, dust to dust – no ‘higher power’ required).

    So, how about it? What if, just if, my little mental masturbation is correct? I understand that I have brutally bastardized a plethora of learned men’s life work, I can’t help it, I just ain’t smart enough to do the math. However, the principles I am describing are FACTS. They are not mere theoretical scribal on paper. They have, for the most part, been objectively verified in experiments that have yielded consistent results to the level that the scientific community was willing to publish many of the ideas AS FACTS and the others as ‘respected theories’ at the very least. Even if my observations are incorrect, inaccurate, and only ‘on the right track’, flat out wrong in may of the details, they nonetheless proceed in an objective and logical form with respect and due regard for the laws of nature as we best understand them today. My thought process sure seems to lead to conclusions that make far, far, far more ‘sense’ than that a guy with a beard did it all in 144 hours; that ‘He’ made man from two hunks of clay, and; that every snowflake is ‘different’ (another invalid assumption based on voodoo instead of a sufficient understanding of chaotic systems and fractals, which, once quantum computing becomes a functional tool, will most certainly be explained) because God makes them that way!

    If you have read this work and, if you have really tried to suspend your ‘belief’ long enough to entertain my effort as an alternative explanation for the nature and origin of the creation of our universe, at least THEORETICALLY capable of replacing your ‘faith based’ responses but just ‘can’t’ or continue to ‘refuse’ to do it, I suggest you get some help.

    If, on the other hand, you can actually get your head around the idea that it is highly more likely to be correct than any of the offered theories of creation that are based on ‘faith’, PLEASE NOTE CAREFULLY, that nowhere, absolutely NOWHERE in here did I say, suggest or imply that there is no God nor that there is no heaven or that there is no ‘afterlife’. However, if you are someday FORCED to swallow the bitter pill that my observations are correct by irrefutable empirical data of the type that freed Galileo, then you are left with no alternative but to face the necessary corollary that regardless of anything else, God, heaven and ‘the afterlife’ are, at best, nothing more than the eventual return of the ‘stuff’ of which we are made to the quantum foam from which ‘this’ particular universe appeared in this form by chance.

    Sorry Albert, but God DOES play dice with the universe.

    David Platek
    perry.m@comcast.net

    • Anon says:

      Is there really such a thing as “nearly infinite”?

    • David:

      I have copied and pasted your comments in italicized bold and then inserted my responses below:

      Sorry to burst all of your ‘bubbles of comfort’, but you have all fallen into (or failed to get out-of) the same trap of circuitous reasoning and boot strapped arguments that you mistake for ‘logic’, all of which continues to take a vast majority of otherwise intelligent and rational humans round-and-round without any real ‘advance’ towards greater enlightenment on a far more important subject, that being, the nature of intelligence, consciousness, and the universe itself.

      David, this opening salvo speaks volumes. By pouring forth so much angry rhetoric (“burst all your ‘bubbles of comfort'”, and “otherwise intelligent and rational humans,” etc.) you have laid bare for all to see the emotional foundations for your atheist beliefs. If you held your atheist beliefs for purely logical reasons, you would not find the need to use angry, forceful rhetoric. Rather, you would just state your views and arguments plainly and calmly.

      The discussion has been and seems to remain just as pointless as when it was first offered for determination as to how may angels can dance on the head of a pin. Although I am not aware of any government grant or ‘faith based’ private investor willing to fund the project, I would be willing to bet my donuts against their dollars that even if all the currently available pins throughout the world could be simultaneously examined by a legion of calibrated and certified instruments employing Non Contact Atomic Force Microscopy (AFM), a new IBM technique that allows imaging down to the resolution of the individual energy bonds that bind atomic structures (See: http://www-03.ibm.com/press/us/en/pressrelease/38856.wss ), not one, not even ONE! ‘angel’ would be found dancing (or doing anything else) on any of the atoms of any of the pins.
      I would also be willing to make a corollary wager that, as I was counting my winnings from the first, the same ‘faith-based-suckers’ whom I had just taken to the cleaners would signify their ‘faith’ by proclaiming that the results of the science ‘didn’t prove that angels don’s exist, only that they must be ‘invisible’ ” or some other similar nonsense. These sorts of responses from the faith based communities, ALL OF THEM, are reflective of a denial that runs so deeply in their minds that most of them are willing to die and send their children to die in defense of the mumbo-jumbo that propagates from their ‘teachings’ and ‘holy books’.

      Once again, more strident rhetoric (“faith-based-suckers”, etc.). And I have to once again point out what I have pointed out to many atheist readers in the past: Strident rhetoric is what one falls back on when one does not have a logically sound argument. One sign of a coherent and logically sound argument is its lack of need to engage in such rhetoric. For example, when Einstein was discussing his theory of relativity with people, do you suppose he needed to say things such as, “Anybody who doesn’t agree with my theory is a sucker or a deluded idiot”? Of course not, because his reasoning was able to stand up on its own, without the need for support from a crutch such as the rhetorical outbursts that you rely upon.

      So I will lead by example and respond to your “angels on the head of a pin” argument in a calm manner without the need for taking rhetorical swipes at you. Since your argument is nearly identical to an argument made by another atheist (Victor Stenger), I will respond by copying and pasting Edgar Andrews reply to Victor Stenger’s argument below:

      Stenger’s first fallacy now lies open to our gaze. He begins (quite correctly) by saying that the existence of God can be addressed using the hypothetic approach. He next labels this approach ‘scientific’ because it is commonly used by science — conveniently forgetting that it is also a methodology applicable to life in general. Thus by claiming that every hypothesis is ‘scientific’, he can assert that the hypothesis of God must be a scientific hypothesis. Finally, he concludes that God doesn’t exist because he cannot be found among the scientific data gathered by microscopes, telescopes or atom splitters. Let’s go over that again. Stenger’s chain of reasoning is as follows [The fallacious statements are italicized]:

      1. The hypothetic approach is applicable to God.
      2. The hypothetic approach is integral to science.
      3. Therefore all hypotheses are scientific hypotheses.
      4. Therefore the hypothesis of God is a scientific hypothesis.
      5. Therefore the hypothesis of God can be tested by scientific experiments.
      6. The data gathered by scientific experiments do not reveal the existence of God.

      Andrews continues:

      A further example of circular argument is the idea promoted by some atheists that ‘science disproves the existence of God’. The assertion is based on the claim that science presents no evidence for the existence of supernatural forces or phenomena. It sounds plausible until you look a little more closely. The argument can be expressed as a syllogism as follows:

      1. Science is the study of the physical universe.
      2. Science produces no evidence for the existence of non-physical entities.
      3. Therefore non-physical entities such as God do not exist.

      Again the fallacy is clear. In point (1) ‘science’ is defined as the study of the physical or material world. This statement thereby excludes by definition any consideration by science of non-physical causes or events. The proposition then argues from the silence of science concerning non-material realities that such realities do not exist. By the same logic, if you define birds as ‘feathered creatures that fly’, there’s no such thing as an ostrich. It’s fairly obvious in this example whose head is in the sand. The correct conclusion, of course, is not that ostriches are mythical but that (on your restrictive definition of ‘bird’) they are not birds. In the same way, to define science as the study of the material universe simply prohibits science from making statements about a non-material entity like God. If the remit of science is deliberately restricted to the physical realm, the fact that science (so defined) tells us nothing about God has no bearing whatever on his existence or non-existence, as most scientists recognize.

      Further, David, your angels-on-the-head-of-a-pin argument against angels (and, by extension, God) rests on the assumption that only that which is material exists (materialism). But we know right away that materialism is false for the simple fact that our universe is imbued with non-physical properties such as time and space. How much time can you find dancing on the head of a pin under a microscope? None…and, by your logic, this implies that time does not exist. Another death knell for materialism is the existence of concepts. Where on the head of a pin can your instruments detect the existence of, for example, a mathematical concept?

      But my reply to your materialism would not be complete without pointing out that modern physics has completely discredited materialism…as I demonstrate in God Is Real…Why modern physics has discredited atheism. A couple citations from that essay:

      Max Planck, the Nobel Prize-winning physicist who founded quantum theory writes:

      “As a man who has devoted his whole life to the most clear headed science, to the study of matter, I can tell you as a result of my research about atoms this much: There is no matter as such. All matter originates and exists only by virtue of a force which brings the particle of an atom to vibration and holds this most minute solar system of the atom together. We must assume behind this force the existence of a conscious and intelligent mind. This mind is the matrix of all matter.”

      The knighted mathematician, physicist and astronomer Sir James Jeans writes (in his book The Mysterious Universe):

      “There is a wide measure of agreement which, on the physical side of science approaches almost unanimity, that the stream of knowledge is heading towards a non-mechanical reality; the universe begins to look more like a great thought than a great machine. Mind no longer appears as an accidental intruder into the realm of matter. We are beginning to suspect that we ought rather to hail mind as the creator and governor of the realm of matter.”
      (italics added)

      I would also be willing to make a corollary wager that, as I was counting my winnings from the first, the same ‘faith-based-suckers’ whom I had just taken to the cleaners would signify their ‘faith’ by proclaiming that the results of the science ‘didn’t prove that angels don’s exist, only that they must be ‘invisible’ ” or some other similar nonsense. These sorts of responses from the faith based communities, ALL OF THEM, are reflective of a denial that runs so deeply in their minds that most of them are willing to die and send their children to die in defense of the mumbo-jumbo that propagates from their ‘teachings’ and ‘holy books’.

      Once again, David, the more you choose to employ strident rhetoric such as “faith-based-suckers,” the more you are going to advertise to third-party viewers of this debate that your arguments cannot stand up on their own. Such rhetoric is only necessary in order to attempt to (1) serve as a crutch for an argument that cannot stand up on its own with bare logic and (2) distract attention from the inadequacy of your arguments.

      Next, I must point out that you are characterizing belief in God as “faith based” despite the fact that this website is replete with logically based arguments for the existence of God. You may want to start by responding to my essay titled God Is Real…Why modern physics has discredited atheism, which I linked to above. Next, you may want to respond to the logically based (not faith-based) arguments for God presented in my essay titled Is There A God? (What is the chance that our world is the result of chance?)

      But, most glaringly, your above comments are posted at the bottom of an essay (about the extreme improbability of our universe emerging by chance) which serves as a logical argument for the the existence of God. Your attempt to characterize belief in God as purely “faith based,” while simultaneously failing to respond to the logically based arguments posted right above your comments, speaks volumes. Very transparently, your tactic is to distract attention from your lack of a counter-argument (rebuttal) to the points made in this essay by instead using a characterization of your opponent’s beliefs in place of a counter-argument. What is your reply to the points made above by Lennox, Hoyle, Gingerich, and Penrose? That they are faith-based even though they are based on math and science?!

      The reality is this: religious ‘solutions’ to problems and any other ‘answer’ to a question that is based upon a ‘belief’ or ‘faith’ in a god or a higher power is nothing more than a placeholder for ignorance. That is, it is a ‘thing’ to plug into the void so that we can get on with our lives. It is a useful artifice that serves a laudable purpose. It is not, however, by any stretch of the imagination, an acceptable ‘answer’ to any question on any one of life’s seemingly endless series of examinations, although, until replaced by a well reasoned scientific alternative, it remains a psychologically necessary one for most. Acceptance of ignorance as an ‘answer’ to anything axiomatically and instantaneously leads to stagnation and puts the brakes on learning. It stops all progress and impedes further development. It leaves a person open and susceptible to harm as a victim of his or own self-imposed ignorance and creates opportunities for snake oil salesmen to continue to peddle their wares. Bowing to the East (or whatever hell direction they bow) at a specific time of day; kneeling, sitting, standing, then knelling then sitting while reading unfathomable phrases in unison; Looking towards the heavens while making the sign of a cross; pointing to the sky after a touchdown; taking one of the now copyrighted ‘Tebow Knee’ poses (I wonder if he is going to bring a class action lawsuit against the Catholic Church to collect royalties – I’m sure he will if it is a part of Jesus’ ‘plan’ for him); pushing the button on the bomb wrapped around your chest ‘for Ala’, and; all the rest of the mindless behavior that continues to this day, is, in my humble opinion, NO DIFFERENT from the ancient ritualistic behavior of religions, cults, and great supposed ‘civilizations’ long past at whose practices we so arrogantly scoff. In fact, I think that the implications for the continuation of such strikingly similar behavior is that, although participants in their respective ‘communities of ignorance’ will disagree, we, as a species, sure don’t seem to have learned very much.

      Here, David, you commit the classic atheist mistake of confusing science and ontology, as I describe in my essays titled Why Trying to Explain Away God With Science is an ERROR and The God of the Gaps: Why God and science are not competing explanations. God is an answer to ontological questions, not scientific questions. By confusing the two, you are committing what is known as a category error in philosophical terms. Science and ontology can and must interact, but science cannot by itself produce answers to such ontological questions as why there is something rather than nothing, or why our universe is so finely tuned produce life, etc… No amount of future scientific research can by itself produce an ontological conclusion. Please recall that both “there is a God” and “there is no God” are ontological conclusions. In other words, theism and atheism are competing ontological explanations, not competing scientific explanations.

      Your arguments are replete with this underlying category error. Science furnishes descriptions and ontology interprets these descriptions. No amount of scientific data can furnish its own interpretation. Below is an excerpt from Who Made God? by Edgar Andrews (as it appears in Why Trying to Explain Away God With Science is an ERROR):

      …science cannot provide complete explanations for natural phenomena. Rather it can only provide useful descriptions. This is why bold declarations from atheists that “science explains things without the need for God” amount to a category error. Edgar Andrews writes in Who Made God?:

      “…far from explaining everything, science actually ‘explains’ nothing. What science does is describe the world and its phenomenology in terms of its own specialized concepts and models — which provide immensely valuable insights but become increasingly non-intuitive as we dig ever deeper into the nature of physical reality.”

      “…The formula [that describes the force of gravity] equates the gravitational force between two objects to the product of their masses multiplied by a universal constant (the ‘gravitational constant’) and divided by the square of the distance between them. But does the equation ‘explain’ why you don’t bump your head on the ceiling? Not really. It tells us there is a force that keeps your feet on the ground, but you knew that already. It also quantifies that force, allowing us to calculate its strength in any particular case, which is extremely useful. But it doesn’t tell us why there is such a force, why it follows an inverse square law, and why the ‘gravitational constant’ has the value that it does. The equation is a description of gravity rather than an explanation.”

      Science describes natural phenomena in terms of laws, but it does not explain where those laws came from, who (or what) enforces those laws, or why the universe has laws in the first place (rather than just chaos). Scientific description, in other words, ends at the level of natural/physical laws. So how does theism explain the above mentioned phenomena? The answer is simple. As I put it in I Believe In Science, Why Do I Need Religion?:

      Such laws are the result of a lawgiver (God). Moreover, theism asserts that matter is nothing more than a manifestation of consciousness (God’s consciousness), which is the view most compatible with modern physics, as I demonstrate in God Is Real: Why Modern Physics Has Discredited Atheism. Robert Boyle, the founder of modern chemistry, summarized the theistic explanation of why matter follows physical laws succinctly when he said: “The nature of this or that body is but the law of God prescribed to it [and] to speak properly, a law [is] but a notional rule of acting according to the declared will of a superior.” [the word “notional” italicized by me]

      Or as James Joule, the propounder of the first law of thermodynamics, for whom the thermal unit of the “Joule” was named, put it: “It is evident that an acquaintance with natural laws means no less than an acquaintance with the mind of God therein expressed.”

      Albert Einstein marveled at the existence of physical/natural laws, and the exquisite order (rather than the chaos that we should a priori expect) which lies therein. He wrote (as cited in The God of the Gaps: Why God and Science Are Not Competing Explanations):

      “…a priori, one should expect a chaotic world, which cannot be grasped by the mind in any way. The kind of order created by Newton’s theory of gravitation, for example, is wholly different. Even if man proposes the axioms of the theory, the success of such a project presupposes a high degree of ordering of the objective world, and this could not be expected a priori. That is the ‘miracle’ which is constantly reinforced as our knowledge expands.”

      Your above 14-point attempted scientific description of the origin of the universe is a textbook example of the confusion of scientific description with ontological explanation. Since your above scientific description references energy that “obeyed the laws of quantum mechanics” (under point #2), I will ask you for your explanation as to why matter and energy follows such laws. The theistic explanation appears above and is further elaborated in the two essays that I mention above.

      I am going to confidently predict that your explanation for why matter and energy follows laws will be a faith-based explanation. How can I be so confident? Because scientific description, as I mentioned above, ends at the level of physical/natural laws. Any explanation must therefore necessarily be extra-scientific, and therefore philosophical/religious.

      In my essay titled I Believe in Science, Why Do I Need Religion?!, I demonstrate that atheism is a faith-based religious belief. One of your atheistic religious beliefs comes to the surface in your above 14-point scientific description. What religious belief am I talking about? The belief that physical/natural laws (such as the laws of quantum mechanics) can be cited as the final explanation for things. Below is an excerpt from that essay which further elaborates on atheist religious belief:

      The God debate is a conflict of religion versus religion, or philosophy versus philosophy…not of science versus religion. ”The so called warfare between science and religion,” writes the eminent historian Jacques Barzun, should actually “be seen as the warfare between two philosophies and perhaps two faiths.”

      What sort of religious beliefs do atheists hold? A prominent religious belief within atheist thought is known by philosophers and psychologists as “scientism,” which holds that the only kind of knowledge that humans can have is scientific knowledge (I also discuss scientism in If the Evidence for God Is So Strong, Why Are So Many Smart People Unconvinced?). Philosopher Mikael Stenmark discusses the problems with this religious view in his book Scientism: Science, Ethics and Religion. As Stenmark points out, the key problem with the premise that “we can only know what science can tell us,” is that this very premise is something that science cannot tell us. It is a self-defeating premise:

      The problem is that the scientistic [not to be confused with “scientific”] belief that we can only know what science can tell us seems to be something that science cannot tell us. How can one set up a scientific experiment to demonstrate the truth of T1 [“T1” is Stenmark’s symbol for the premise, “The only kind of knowledge that we can have is scientific knowledge.”] What methods in, for instance, biology or physics are suitable for such a task? Well, hardly those methods that make it possible for scientists to discover and explain electrons, protons, genes, survival mechanisms and natural selection. Furthermore, it is not because the content of this belief is too small, too distant, or too far in the past for science to determine its truth-value (or probability). Rather it is that beliefs of this sort are not subject to scientific inquiry. We cannot come to know T1 by appeal to science alone. T1 is rather a view in the theory of knowledge and is, therefore, a piece of philosophy and not a piece of science. But if this is the case, then T1 is self-refuting. If T1 is true, then it is false. T1 falsifies itself.

      Regarding Stenmark’s above comments, just think about it: How could a statement such as, “The only kind of knowledge that we can have is scientific knowledge,” be verified scientifically? With a chemistry experiment utilizing a bunsen burner and test tubes? With a physics experiment utilizing a particle accelerator? Because the belief that, “The only kind of knowledge that we can have is scientific knowledge” CAN NEVER ITSELF BE SCIENTIFIC KNOWLEDGE, it is a self-refuting belief.

      Einstein surely understood that scientific knowledge cannot be the only kind of knowledge, and that it must necessarily interact with religious / philosophical reasoning…which is why he said, “Science without religion is lame, religion without science is blind.” So did many other crucial contributors to modern science…such as Max Planck (the Nobel Prize winning physicist who founded quantum theory), which is why he said, “There can never be any real opposition between religion and science; for the one is the complement of the other.”

      Finally, I must address your contention that belief in God is the product of underlying psychological motives: In If The Evidence for God Is So Strong, Why Are So Many Smart People Unconvinced, I lay down the psychological motivations for belief in atheism. Atheism is psychologically motivated by the need to do away with the repugnance that some people feel towards the concept of being subordinate to a higher power. We can see the repugnance that you feel towards this concept manifested in your angry rhetoric. Your sword is double-edged.

      • Dalton says:

        Scott,
        I am reading your essay, I’m not to far into it but I have a question based on this quote:

        “In the New Story of science the whole universe–including matter, energy, space, and time–is a one-time event and had a definite beginning. But something must have always existed; for if ever absolutely nothing existed, then nothing would exist now, since nothing comes from nothing. The material universe cannot be the thing that always existed because matter had a beginning. It is 12 to 20 billion years old. This means that whatever has always existed is non-material. The only non-material reality seems to be mind. If mind is what has always existed, then matter must have been brought into existence by a mind that always was. This points to an intelligent, eternal being who created all things. Such a being is what we mean by the term God.”

        I follow this and it makes sense up to a point… But if I’m not mistaken, mind can’t create anything without something. Mind would require matter in order to create a new matter, so this doesn’t answer the question of how anything started. Sure there could be a god like mind overseeing all things, but without building blocks nothing could have been built.

        • Scott Youngren says:

          Dalton,

          Don’t feel bad if this stuff is hard to wrap your head around because it is difficult even for elite physicists to wrap their heads around. Niels Bohr, one of the most important physicists of all time, said, “Anyone who is not shocked by quantum theory has not understood it.” Make no mistake, this stuff is mind-bending.

          It is important to understand that the insights of modern physics require a complete re-frame of the way we look at the world. This re-frame can be compared to the re-frame which was necessary when, many hundreds of years ago, people began to understand that the Earth is round, and not flat. Image how difficult it was for people to accept that their perceptions of the world were wrong. A person being told that the Earth is round for the first time may have become confused and asked questions such as, “How can that be? Why doesn’t everything and everybody just fall off, then?”

          The re-frame of our world which modern physics requires us to make is this:

          Matter, as it appears to us, is an illusion. I discuss this topic in depth in God Is Real: Why Modern Physics Has Discredited Atheism. Some relevant excerpts from that essay:

          Physicist Richard Conn Henry, from Johns Hopkins University, writes:

          “Why do people cling with such ferocity to belief in a mind-independent reality? It is surely because if there is no such reality, then ultimately (as far as we can know) mind alone exists. And if mind is not a product of real matter, but rather is the creator of the illusion of material reality (which has, in fact, despite the materialists, been known to be the case since the discovery of quantum mechanics in 1925), then a theistic view of our existence becomes the only rational alternative to solipsism.”
          [“Solipsism” is defined as “the view or theory that the self is all that can be known to exist.”]

          Max Planck, the Nobel Prize winning physicist who founded quantum theory said:

          “As a man who has devoted his whole life to the most clear headed science, to the study of matter, I can tell you as a result of my research about atoms this much: There is no matter as such. All matter originates and exists only by virtue of a force which brings the particle of an atom to vibration and holds this most minute solar system of the atom together. We must assume behind this force the existence of a conscious and intelligent mind. This mind is the matrix of all matter.”

          The knighted mathematician, physicist and astronomer Sir James Jeans writes (in his book The Mysterious Universe)…

          “There is a wide measure of agreement which, on the physical side of science approaches almost unanimity, that the stream of knowledge is heading towards a non-mechanical reality; the universe begins to look more like a great thought than a great machine. Mind no longer appears as an accidental intruder into the realm of matter. We are beginning to suspect that we ought rather to hail mind as the creator and governor of the realm of matter.” (italics added)

          I highly recommend that you read my essay God Is Real: Why Modern Physics Has Discredited Atheism and watch the embedded video. You may also want to read The Matter Myth by physicists Paul Davies and John Gribbin, in order to get a better grasp of this topic. If you want to save time, you can just read the article titled Mental Universe by Johns Hopkins University physicist Richard Conn Henry, to which I provide a link in the above mentioned essay.

          You have probably seen the movie The Matrix. Recall that in this movie, what people perceive as the physical world is really just an illusion of a physical world created by a computer program. Well, our reality is not an illusion created by a computer program, but rather, it is an illusion of material reality created by an eternally existent consciousness (God). But God did not dupe us into thinking that the world is material…our culture did.

          Scott

    • Graceus says:

      David,

      The first half of your entire comment is a complete red herring fallacy. More accurately, it’s Bulverism, and it’s interesting that many atheists will start off with Bulverism when commenting on this site, possibly to tilt the discussion in their favor before even getting to the actual argument. But since the first part of your comments have nothing to do with the actual argument, no tilting in their favor is accomplished. So, when you use the words “pointless” and a “‘waste of otherwise useful intellectual effort”, those words apply to the first part of your comments that have nothing to do with the topic at hand, which is: “What is the probability the universe is the result of chance?” Also, just because people have been wrong in the past, does not mean that they will always be wrong in the future. By your logic, we could say that since scientists have been wrong in the past, they will therefore always be wrong in the future with their scientific theories and experiments. That’s illogical.

      What I find contradictory about your comments is that you state “I do not profess to have an ‘answer’ as to whether there is a god or a creator, and I think any debate at this stage of our collective ignorance wastes otherwise useful intellectual effort…”, but then go on to assert that there is no God or creator in the first point of your description of the universe, even belittling belief in God, and then repeat in your conclusion “nowhere, absolutely NOWHERE in here did I say, suggest or imply that there is no God.” If you don’t want to suggest or imply that there is no God, then don’t do it.

      Taking a look at your 14 points, you keep mentioning Time, which is irrelevant. Christians have philosophical arguments for the origin of the universe that can be used for A or B Theory of Time. A versus B Theory of Time is purely philosophical.

      “The universe in which we live came into existence as a consequence of the interaction of googolplex upon infinite googolplex of strands of pure energy (‘strings if you like’) that obeyed the laws of quantum mechanics, existing in all possible energy states simultaneously; being in all possible dimensions simultaneously; exhibiting maximal entropy, completely devoid of any form, wholly ‘unobserved’, free from decoherence but ‘boiling’, ‘bubbling’ and ‘alive’ with the constant emergence of a particle and an arrow of time followed by the disappearance of both within one plank length, in any one of any and all of the dimensions (the quantum foam).”

      Am I correct that you theorize the universe came into existence by itself because of Quantum Foam? According to “Ask The Astronomore” web site, “The problem is that we have no evidence that 1) gravity is a quantum field and 2) that space-time has this type of structure at these scales” (http://www.astronomycafe.net/qadir/ask/a11792.html). Also, with regards to the quote above, what is your source for that? I hope you are not plagiarizing. When a footnote is attached to a quote (which we see later in your points), that informs me that these are not your own words, and you need to cite where your information came from.

      What is interesting in your points are the references to laws when the word ‘govern’ is mentioned: “…interactions, the formation of the particles, the arrows of time, the dimension they choose, are all _governed_ by nothing more than the probability of their occurrence” and here “the foam yields maximal entropy to the _rules_ that _govern_ it”. This is an assumption (and also why science cannot be justified by the scientific method; science is permeated with unprovable assumptions!) – the assumption that accidental acts could give rise to order and rules, and the laws of nature. Please read Scott’s comments again that begin with “Science describes natural phenomena in terms of laws, but it does not explain where those laws came from, who (or what) enforces those laws, or why the universe has laws in the first place (rather than just chaos).” I’m also going to quote a fellow apologist who commented on this site not too long ago:

      “This is the bootstrapping problem that I find to be a big problem is on the side of naturalists. Cf: Dr. Craig: ‘It’s a metaphysical bootstrapping problem. It’s like Escher’s two hands that draw each other. On the one hand, the laws of nature are to account for the existence of the world. And, on the other, you don’t have a law unless you have a world for those laws to be laws *of*’.” If you know anything about the Big Bang, you would know that the _evidence_ shows that everything in the universe (which includes the laws of nature) traces back to the initial singularity. Anything existing beyond that (such as the laws of nature) is a metaphysical or philosophical question.

      I have to take issue with your description of particles here: “…fell under the watchful eye of the ‘particle’ and were ‘forced’ to make a ‘choice’ as to what state they wished to take. Some chose to be ‘the same’ and became entangled. Some ‘chose’ other dimensions. Still others ‘chose’ to be in the third dimension but took on another energy state.”

      Since when did particles have characteristics of an intelligent being where a particle can observe things and have choices? As far as I know, the only things that can do that are intelligent beings. It is as if you want to do away with a personal, intelligent being – God, but want to be able to assign characteristics of a personal, intelligent being to things such as a particle. Is that logical? If a particle actually cannot see things and have choices, as if the particle actually has a mind to make decisions from, then don’t give it characteristics of an intelligent, personal being. There seems to be signs of intelligence within our universe, and yet atheists deny the source-God, the intelligent, omnipotent, personal Being.

      -Grace

      • David Platek says:

        Dear Grace,

        Thank you for responding to my post. I especially want to thank you for your observation that the first half of my comment struck you as being Bulverism. Although a potentially cogent observation, I couldn’t help but observe that it’s credibility was instantaneously destroyed with the phrase, “interesting that many atheists will start off with Bulverism when commenting on this site”, words which, in and of themselves, present the very fallacy of which you complain by introducing the irrelevant ‘red herring’ subject of what ‘many atheists’ may have done in the past in posting to the site. This introduces the wholly irrelevant necessary implication that I am an atheist through the use of another fallacy; affirming the consequent (“atheists employ Bulversim” which leads to: “I have concluded the first half of the comment to be Bulverism”, therefore: “you must be an atheist”). Finally, with all of that in place, you are then able to close the loop and ‘prove’ that the first half of my comment is, ‘in fact’ “Bulverism” through the simple application of the final necessary fallacy of begging the question that goes something like this: The poster of the comment is an atheist; atheists often start off with Bulverism when commenting on this site, therefore; the first half of the entire comment can be dismissed as ‘Bulverism’. See how easy and convenient that was?
        Now that we have that out of the way, can we both at least agree, in principle, that the human animal has a strong tendency to engage in fallacious arguments? And that, even when the intent is wholly and completely otherwise, a careful critic can almost always dig deeper and deeper and deeper into any set of words or phrases to ‘expose’ fallacies in the argument to the point that the process of discourse itself breaks down into a meaningless exercise in semantics and becomes a ‘fallacy’ of a sort onto itself?
        So, if you would give me some ‘wiggle room’, I will return the favor in kind and we may proceed to, as you suggest, address the topic at hand: What is the probability the universe is the result of chance?
        In reply to:
        “Also, just because people have been wrong in the past, does not mean that they will always be wrong in the future. By your logic, we could say that since scientists have been wrong in the past, they will therefore always be wrong in the future with their scientific theories and experiments. That’s illogical”.
        I offer you a resounding ‘yes’ to the first sentence and cry ‘fallacy foul’ as to the remainder (See, discussion above). Although your reply is non-specific to my comment, I am presuming that you are making reference to those portions which illustrate that, throughout the recorded history of humans on the planet Earth, faith based ‘answers’ to all manner of inquiry have always been proven to be WRONG, 100% of the time, once the scientific community has collected sufficient data to publish, thereby removing the matter from the realm of philosophy and replacing the previous ‘faith-based-answer’ with one based upon scientific ‘fact’ (or, at least a pretty darn-good-theory).
        Please note however, that in making these sorts of statements, I do not intend to attack ‘faith based solutions’ as ‘wastes of time’ or ‘meaningless exercises’, I am only commenting on the objective reality that: People used to be put in jail for ‘blasphemy’ (actions defined as ‘criminal’ by men, taken against the then current concept of a ‘deity’, also created and defined by men, and/or their proclamations as to the ‘truths’ of the faith-based-community as it existed at any given point in time). Galileo is only one of many examples of the FACT that these things occurred and no amount of fallacious reasoning could dispute this reality, right? Well, what happened? Eventually, the emerging and ever evolving field of science learned, through observation and experimentation, coupled with intellectual discourse amongst learned men, MOST OF WHICH by the way, were and remained devout in their religious beliefs, collected data and empirical evidence that began to cause the emergence of alternative explanations in the form of ‘scientific theories’, many of which were opposed to one another, that eventually evolved into verifiable ‘proof’ of a ‘reality’ that was so convincing and undeniably more correct than the ‘faith-based-proclamation’ that ‘God’ had made the heavens revolve around the Earth, that Galileo was ‘pardoned’ for the ‘crime’ of having had the foresight to replace ‘ignorance’ with a better understanding. Never mind that it took the Catholics until 1992 to do it (http://4thefirsttime.blogspot.com/2007/09/1992-catholic-church-apologizes-to.html), at least they did it, right? You see, ‘faith’ had no choice but to yield to science because the scientific ‘answer’ was supported by a logical, systematic and verifiable set of algorithms that left so little room for ‘doubt’ as to their validity that continued insistence to the contrary would not only ‘illogical’, but irrational as well.
        This ‘phenomenon’ of systematically replacing ‘faith-based-solutions’ with those born of better reasoning producing a superior result is patently obvious. Its progression and evolution throughout history can be easily mapped and is impossible to refute with anything other than a fallacious argument. However, please further note that even when ‘science speaks’ it too does so with a certain authority of ignorance because nobody, but nobody, really “knows” anything at all. As far as I am aware, there is no credible scientist on the planet Earth (operative word: “credible”) who will declare or state a proposition with 100% certainty. We have seen that Newtonian physics has its limitations and that the ‘answers’ generated by the equations we were all tested on for accuracy in high school are actually ‘wrong’, even in our own experience, but the ‘error’ is so small that you can ignore the effects of the warping of space-time by gravity, even for the purposes of building buildings and rocket ships to the moon. However, if we learn to travel at or near the speed of light, Newtonian ‘answers’ to the equations must be thrown into the waste basket in favor of the higher and better understanding of the Universe as described by Einstein.
        So yes, Grace, to more directly answer even the ‘objected to’ portion of your question: qualifiedly and subject to the phenomenon described above – ‘yes’, every answer provided by men ‘of science’ will carry with it, a certain amount of ‘ignorance’ and, consequently, will be correspondingly ‘wrong’, and remain so, until such time as a new and higher level of understanding comes along to better explain, refine, re-define, augment or replace it altogether. At any instant in time however, it has ALWAYS BEEN TRUE, that a well reasoned scientific response to an inquiry presents a better alternative in making a choice as to how to proceed than the ‘faith-based-solution’ that it has replaced. For example: If your child was struck by fever of 103°, you would call a doctor, right? You may also chose, to say a few prayers, you know, to cover your bet. However, if you did NOT pursue the ‘scientific based’ set of solutions by calling the doctor and relied solely upon prayer, most states (and most other countries around the world) would exercise subject matter jurisdiction over you and your child and you would find yourself in jail if your child should succumb and die as you continued to pray over the body. Now I am not suggesting (which would be a wholly fallacious argument) that we have come ‘full circle’ and ‘god’ has ‘lost to science’ (Galileo (‘then’) and You only praying (‘now’)) or some other such nonsense, but what I am suggesting is that these FACTS provide a basis upon which I can confidently state the phenomenon I claim to ‘observe’ is real rather than perceived and most certainly NOT the product of ‘Bulverism’.
        Moreover, and even more resoundingly, YES, YES, YES, the scientific community is always, continually, openly and freely admitting that it is or has been ‘wrong’ about something, provided only, that the purported replacement ‘tearing down’ the previously positioned ‘fact’ passes the same or greater degree of scientific scrutiny than that which was applied to the proposition under attack. By this method, we obtain at least a reasonable degree of certainty that science, that is ‘good, well reasoned science’, the kind upon which an overwhelming majority of rational persons will agree, will not become inundated with and diluted by ‘hocus pocus’ and ‘slight of hand mumbo-jumbo’. What I find most fascinating is, that when anyone even suggests to a member of the faith-based-community that their then current notion of ‘god’ might have a few issues squaring with reality because we may actually have come into existence from nothing by random chance and/or that this universe may be nothing more than a holographic projection of information existing in another dimension, and so on and so on, the troops circle the wagons and ‘hunker down’ into ever increasingly small circles of circuitous reasoning, launching salvo after salvo of fallacious argument instead of simply taking pause to consider that maybe, just maybe, it’s time to ‘evolve’ and take a position of higher, deeper, and richer understanding, not abandoning ‘faith’ but simply removing however many placeholders of ignorance are required to be removed in order to ‘make room’ for the better answer.
        Now, you also claim to have found some sort of ‘contradiction’ in my comments in the following terms:
        “but then go on to assert that there is no God or creator in the first point of your description of the universe, even belittling belief in God, and then repeat in your conclusion “nowhere, absolutely NOWHERE in here did I say, suggest or imply that there is no God.” “.
        All I can say to you in response to your claim is that it is wrong. The ‘first point’ of my theory, my idea, my offered for consideration and purely intellectual exercise, item numbered ‘1’, is clearly and conspicuously preceded by the following sentence: “Try to imagine that the reality IS that:” Either you skipped over these words or you are compelled to dismiss them so you can indulge your apparent proclivity for yet another fallacious argument. Either way the only thing I can say, once again, is that there is no ‘contradiction’ in my comment of the sort you purport to describe. Moreover, I take issue with your assertion that I have somehow ‘belittled belief in God’. You are free to believe in anything you wish to believe in. If my words have caused you to get that impression, I’m sorry, and I can assure you that such was not my intent. I have read my comment again, looking for portions that could be taken in the manner you suggest, and I see how you might come to that conclusion if you have failed to appreciate my sometimes crude attempts to paint a metaphor or merely make a point. All I can offer in this regard is my apology together with the further invitation for you to read my effort again and see if, and to what extent, your reaction to my words may have clouded your ability to see the message they were intended to convey.
        Next you write: “[t]aking a look at your 14 points, you keep mentioning Time, which is irrelevant”. Wow! That is a tough one to deal with. I can’t imagine anything that doesn’t have something to do with time. I think every philosopher as well as ever scientist on the planet has some notion of ‘time’ and understands that a better understanding of ‘time’, both in the scientific and philosophical realms, is essential to a better and higher understanding of the Universe. Obviously, you don’t really mean what you say, right? I mean, if you are going to bake a cake or watch television, or even have a memory of anything that happened in your life ‘in the past’, you have to rely on the existence of ‘time’, don’t you? If not, please explain why you can remember your name or how you ever get through ‘the next day’?
        Assuming the more likely case that you have not, as yet, paid much attention to the consideration of time, or, that you are confused about time, or, in the further, and equally likely case that you have blindly accepted some sort of ‘faith-based-proclamation’ as to the nature of time and are either unwilling or afraid to explore the concept further, I urge you to take some ‘time’ (pardon the pun – again) to explore these links which may give you some insight beyond your present understanding:
        http://en.wikipedia.org/wiki/Arrow_of_time
        http://q2cfestival.com/play.php?lecture_id=7731
        I don’t want to get into any more detail than to just say I’m sorry if your faith insists on ‘keeping you in the dark’ so to speak, by clinging onto notions that are simply no longer valid. I am really NOT trying to ‘pick a fight’, but please, please, please consider that maybe, just maybe, we are all engaging in no more than a continuation of the same type of ‘intellectual arm wrestling’ that must have gone on during the time of Galileo. Maybe, just maybe, Christians, as well as each and every other organization bound only by ‘faith’ is struggling (and rightfully so) to ‘hang-onto’ its very existence out of ‘fear’ that if all the icons and ideology are replaced by higher and better reasoned explanations, there will be nothing left to ‘believe-in’, at which point, the comfortable numbness provided by blankets of ignorance will all be folded and put away leaving nothing but a cold hard ‘reality’. While I too much prefer my early childhood, one that was safe, comfortable, and predictable in all that I could perceive, my inevitable growth, learning, and intellectually maturity, all driven forward by the collapsing wave generating the relentless arrow of time, forced me to abandon my childhood ‘truths’ and ‘beliefs’ (Santa Claus, Easter Bunny, Tooth Fairy, that Japan ‘snuck-up-on-us’ at Pearl Harbor, that Oswald acted alone, etc.), in favor of the often times more cold, harsh and unpleasant yet undeniable realizations that replaced them. All I can do is encourage you to explore the links above, as well as any other to which they may lead you if you are so inclined, and then consider whether it may be ‘time’ to at least reconsider your statement that, “[t]ime is purely philosophical”.
        As to your inquiry as to whether I theorize that the universe came into existence by itself “because of Quantum Foam”. My answer is predominately ‘yes’. It did not arise ‘because’ of the quantum foam, but my idea is that it arose ‘from’ the quantum foam, as a consequence of the application of the laws of probability, quantum mechanics, entropy and, most likely other as yet ‘unknown factors’ all of which currently lie currently within the realm of philosophy and ‘faith-based-solutions’ (see, plenty of room left for ‘god’). Moreover, I agree with the pronouncements that there is little to no current evidence for gravity existing as a quantum field, although there is a whole bunch of math that seems to indicate such may be the case. In addition, I wholly concur that no evidence has been found to support the notion that space-time has the suggested structure of a ‘quantum foam’ at ‘these scales’ (referring, I presume, to the Plank scale). In fact, results of recent experimental observations conducted by NASA comparing the arrival time of high energy photons with that of photons having more than a million times less energy leaving a Super Nova simultaneously (several billion years ago) failed to produce the predicted ‘delay’ that was expected had they traveled ‘through’ a ‘quantum foam’. The postulate under consideration was that, if space-time were, in fact, made of a ‘quantum foam’, the higher energy photons should have ‘interacted more’ with the foam along their journey to Fermi and, as a consequence, have been ‘slowed’ by the interaction and arrived at a measurable time ‘after’ their less ‘energetic’ cousins. The failure to detect any significant difference in the arrival times is currently being offered as ‘proof’ of the non-existence of ‘quantum foam’ and even as ‘proof’ that space-time is smooth, as suggested by Einstein (See, http://www.space.com/15297-gamma-rays-prove-einstein-space-time-smooth-video.html ).
        However (please pay extra close attention to this part), these ‘conclusions’ have not been ‘published as fact’ by the scientific community because they represent but one data in the search for an ‘answer’ to the question as to the nature of space-time. Because the ‘score’, if you will, in the ‘game’ of these observations is ‘one-to-nothing’ at this time, but the fat lady has yet to sing on this one. The fact of the matter is, that the Casmir Effect (http://en.wikipedia.org/wiki/Casimir_effect ), a well known and generally accepted scientific principle that has been verified over and over and over again by controlled laboratory experiments, has conclusively established that something exists in otherwise ‘empty space’ that causes two plates placed in close proximity in a vacuum to be ‘squeezed’ together. It has long been generally accepted that the ‘force’ pushing the plates together is the result of ‘wave pressure’ generated by matter-antimatter between particles and ‘anti-particles’ that spontaneously ‘appear’ out of nothing and then ‘vanish’ back into ‘nothing’ – the Quantum Foam. These events occur everywhere with the same probability of occurrence. Those that produce waves short enough to ‘fit between the plates’ have an equal affect on them, occurring in roughly the same numbers on both sides of each plate. However, those events that try to occur between the plates with a wavelength too long for them to ‘fit’ between the plates cannot happen between the plates, creating an imbalance in the ‘pressure’ of the sum of all of the waves with the higher ‘pressure’ being outside of the plates, causing them to be ‘pushed’ towards one another. This effect has been well studied and is so well known that it has been accepted as ‘strong evidence’ of the existence of a ‘quantum foam’. So, unless and until a better, more credible, more thoroughly tested and understanding explanation is offered for this effect, it would be scientifically irresponsible to declare from the NASA Fermi observations to the contrary that: space-time has been proven to be smooth and that the Quantum Foam ‘does not exist’. In fact, if you chose to watch the video linked to the ‘citation’ in the foregoing paragraph, you will note that it DOES NOT appear on the NASA website (or at least I haven’t been able to find it anywhere there) and that the ‘conclusions’ drawn by the researcher seem to be hers and hers alone. I have looked for confirmation of her assertions from other sources and, so far, it seems that these results, rather than ‘proving’ or ‘disproving’ anything stand as no more than data that will require far more investigation and a long long way to go before it is entitled to being moved ‘up the food chain’ towards acceptance as ‘fact’.
        As to the rest of it all, I know, understand and can appreciate how difficult it is for anybody to get their head around the notions that ‘things can appear from nothing’, as well as most, if not all, of the other phenomenon associated with investigating the ‘world of the very small’ at the Plank length. I am a comparative moron and can’t even begin to do the math, but the existence of these principles cannot be denied. Even as hard as he tried, Einstein, before his death, was compelled to accept, as a FACT, and NOT as a matter of opinion, that quantum mechanics was real and that it obeyed certain rules and laws which governed how particles and waves behave. If you are really interested in learning more about the world in which you live, please take some time to explore the information attached to these links:
        http://en.wikipedia.org/wiki/Quantum_mechanics
        http://en.wikipedia.org/wiki/Quantum_entanglement
        http://www.youtube.com/watch?v=s3ZPWW5NOrw&feature=related
        http://www.youtube.com/watch?v=DfPeprQ7oGc
        http://en.wikipedia.org/wiki/Entropy
        http://science.discovery.com/tv-shows/wonders-with-brian-cox/videos/wonders-of-the-universe-entropy.htm
        http://en.wikipedia.org/wiki/Schr%C3%B6dinger's_cat
        http://www.youtube.com/watch?v=IOYyCHGWJq4

        Now, onto a little more detailed and direct response to your reply. You ‘dismiss’ as an ‘unprovable (sic) assumption’ that “accidental acts could give rise to order and rules, and the laws of nature”, but that is, precisely, what the proven to be correct 100% of the time so far ‘discipline’ of quantum mechanics clearly and most convincingly indicates. Please, please, please take some time to learn about the quantum properties of matter by exploring the double slit experiment. Then, if you are still in the game, follow the implications of the Schroeder’s Cat thought experiment. If you still have anything left in the tank, open the link above giving you a definition of quantum entanglement and then give these a try:
        http://www.youtube.com/watch?v=0Eeuqh9QfNI
        http://www.pbs.org/wgbh/nova/physics/spooky-action-distance.html
        And if you watch the little video appearing at: http://www.youtube.com/watch?v=QErwOK3S5IE&feature=related , which is a nice short presentation by Dr. Michio Kaku explaining entanglement in elementary terms (perfect for me), you will note that he ‘postulates’ that any ‘information’ that might be sent by way of the use of the phenomenon is ‘most likely’ going to be ‘useless’, an observation that allows him to conclude that Einstein ‘may’ as yet have the last laugh. However, and contrary to his muse, physicists have more recently used the properties of quantum entanglement to transport photons over 88 miles, instantaneously, from one physical location to another (See, http://www.forbes.com/sites/alexknapp/2012/09/06/physicists-quantum-teleport-photons-over-88-miles/ ). Proving, in my opinion, that even within the most respected arena of the scientific community, nobody holds onto ideas so tightly as to foreclose the possibility that, regardless of their conviction in their ideas, unless supported by data sufficient to raise an idea to the level of a ‘fact’ (or at least, the best repeatedly objectively verifiable ‘theory’ so far), nobody, absolutely nobody, would stand in the way of proof to the contrary. Even at the end of his life, Einstein continued to hold onto his religious beliefs but had no choice but to accept that quantum mechanics – including entanglement, was real. How he finally reconciled the obvious resulting conflict in his own mind is known only to…..well…. (insert any convenient placeholder for ignorance ‘here’) ‘god’.

        Q. So, how can something come from nothing?
        A. By the inevitable probability of the event taking place.

        Q. Where did the ‘laws of nature’ come from?
        A. Other than those that existed prior to the singularity, they were all defined within the inflationary period that followed, when matter first took form and the process of the journey back towards maximal entropy began. We ‘exist’ as we ‘exist’ because and only because of the occurrence of those events in the first instant following the singularity. They occurred purely by chance, developing into a highly complex chaotic system (See, (http://en.wikipedia.org/wiki/Chaos_theory, http://sprott.physics.wisc.edu/chaos/comchaos.htm ) that determined and established the rules for what we currently describe as: gravity, the strong force, the weak force, all the sub-atomic particles identified to date, dark matter, dark energy, and; whatever the heck else there is that we are capable of discovering in the future.

        Q. Since when did particles have characteristics of an intelligent being where a particle can observe things and have choices?
        A. Ever since they came into existence in our three dimensional universe. Please note that I am NOT claiming particles to be ‘beings’ of any sort, intelligent or otherwise. But they DO possess the ‘characteristics’ of intelligence in that, when ‘observed’, a quanta of mass (a ‘particle’) will be forced to ‘assume’ one of all of its possible ‘states’, an event characterized as being a ‘choice’ made ‘by the particle’ rather than an ‘effect’ imposed upon it by the observation. Also, the act of ‘observation’ in quantum mechanics is NOT intended (although in more complex systems may have a butterfly effect on the system as well) to mean actual viewing by a pair of human eyes. Rather, it connotes ‘measurement’ or ‘detection’ or any other method of attempting to ‘learn’ or ascertain ‘anything’ about the subject of the observation. The ACT of engaging in the attempt to gain information itself instantaneously causes the effect of ‘influencing the outcome of the observation’ in some way. In quantum mechanics, this is known as ‘decoherence’ (See, http://en.wikipedia.org/wiki/Quantum_decoherence ). In other disciplines, a similar effect is known as the Uncertainty Principal which states that you cannot know both the position and momentum of a particle at the same time, and, that the more certain you become about one, the less certain you become about the other (See, http://en.wikipedia.org/wiki/Uncertainty_principle ; http://en.wikipedia.org/wiki/Observer_effect_(physics) ).
        Although I have no citation for you to support my idea that this effect exists at the Plank scale between ‘stings’ of pure energy and a collection of two or more of them (something that is as yet ‘unproven’ to exist), it seems logical to presume such to be the case. It has been shown that even the slightest interaction between any ‘stuff’ massive enough to produce an output capable of being captured as ‘data’, even as ‘small’ as a pulse of light from a laser, WILL, more often than not, cause an electron or even things as small as a single photon to be ‘forced’ to assume a particular state to the exclusion of all other possible states (make a choice). I can see no reason why the effect does not obey some scalar function so that it would remain viable in all directions within space-time. Although it might be expected to decline at logarithmic scales as systems grow in complexity, if, for no other reason than the sum of the wave functions involved in systems growing in complexity past one or two hundred cubits (a ‘quanta’ of anything, like a single photon) would yield correspondingly declining probabilities that the entire system would respond measurably to ‘observation’, even at the macroscopic level (which could be the ONLY REASON why we ‘stay put’ so to speak, and don’t simply ‘disappear’ into another dimension from time-to-time), but the effect would most likely grow along the same scale as we continue to discover more massive ‘particles’ existing at even higher energy levels and occupying smaller and smaller areas of space, down to what I believe to be a singular quanta of pure energy occupying no more than one (1) plank length of space and which would posses a nearly ‘infinite’ amount of energy (or at least the maximum allowable amount – more ‘rules’ yet to be discovered) and, consequently, no mass, exhibiting ONLY the properties of a wave.
        At such a level of ‘existence’, the definitions of concepts such as ‘intelligence’ and ‘beings’ and all the rest would be forced to yield to the realization that the universe consists of nothing but energy, in its purest form, that obeys only the second law of thermodynamics as well as the laws of quantum mechanics and their attendant probabilities. The ‘quanta’ of all rules that are, just because they ‘are’, and nothing more. Leaving the potential for further exploration of what would then be the only matter remaining for debate: from ‘where’ did all the energy ‘come’. A concept that would make no sense whatsoever in the universe and have no meaning whatsoever once maximal entropy had been re-established because, in order to have a question involving a ‘present’, you have to have a sense of a ‘past’, which, of course, requires that time VIOLATE the laws of nature and continue to exist beyond the collapse of its own wave into the quantum foam. Sorry, but that ‘ain’t gonna happen’, and the inquiry, as well as the universe itself will come to an end.

        Finally, if you have fifty-two-minutes and four seconds left, I urge you to pour yourself a nice glass of wine, sit back, relax, and watch this: http://www.youtube.com/watch?v=EjaGktVQdNg&feature=related

        Thanks for the opportunity for me to exercise my brain a little bit.
        Dave

        • David:

          Could you do us all a favor and try to condense your arguments? Is this an attempt to throw people off with quantity of argument in order to distract from the lack of quality of argument?

          After skimming your comment (which is absolutely huge), I am going to reply to what is (as best as I can tell) the substantive, non-rhetorical portion of your comment. If I have missed any of the substantive portion of your argument, please point it out to me. And also please try not to use such massive amounts of verbiage to make your point. It is very likely that intelligent third party viewers of this discussion can recognize the tactic of trying to disguise a weak argument within large amounts of verbose text.

          You write:

          Q. So, how can something come from nothing?
          A. By the inevitable probability of the event taking place.

          I have replied to this argument in my essay titled Why God? Why Not Just Plain Luck? Probability needs something to act upon. There is no such thing as probability with nothing for that probability to act upon. An excerpt:

          Sure—the atheist argument goes—the probability of such things occurring naturally is very low…but with enough time, and even a slight probability, what is there to prevent virtually anything from happening?! But this atheist reasoning makes some very grave oversights. First of all, bare probability and large amounts of time, alone, cannot accomplish anything, ever. Period.

          Does this seem like too strong a statement? It is not. Probability and time can never accomplish anything without 1) a causal mechanism and 2) an underlying structure, or order.

          As an illustration, consider the lottery: Even though the chance of a specific individual winning the lottery is incredibly small, many people have won lotteries in the past, and many more will win in the future.

          What is necessary, then, for this bare probability of a lottery win to result in an actual lottery win? Much more than just time and chance. For one, in order to win the lottery, the causal mechanism of going to the store to buy lottery tickets on a regular basis is required. Without this causal mechanism, the probability of winning the lottery is exactly zero. As one lottery advertisement says, “You can’t win unless you play.”

          Secondly, the structure of a lottery commission and a distribution network for lottery tickets (etc.) is necessary (not to mention the monetary system of the Dollar, Euro, etc.). In other words, for one to win the lottery, there must first be a lottery, and there must first be such a thing as money to win.

          Further, you seem to have overlooked the fact that “inevitable probability” is something, not nothing. How could you make this oversight? A state of nothingness also necessarily implies a state of no probability.

          Next, you write:

          Q. Where did the ‘laws of nature’ come from?
          A. Other than those that existed prior to the singularity, they were all defined within the inflationary period that followed, when matter first took form and the process of the journey back towards maximal entropy began. We ‘exist’ as we ‘exist’ because and only because of the occurrence of those events in the first instant following the singularity. They occurred purely by chance, developing into a highly complex chaotic system (See, (http://en.wikipedia.org/wiki/Chaos_theory, http://sprott.physics.wisc.edu/chaos/comchaos.htm ) that determined and established the rules for what we currently describe as: gravity, the strong force, the weak force, all the sub-atomic particles identified to date, dark matter, dark energy, and; whatever the heck else there is that we are capable of discovering in the future.

          Once again, David, there is no such thing as chance acting on nothingness. This is patently absurd. Recall my above illustration: Unless there is first a lottery and a monetary system, the probability of anyone winning the lottery is exactly zero. Further, probability itself is something, not nothing. For further edification, please read this New York Times book review of A Universe From Nothing written by the atheist physicist Lawrence Krauss. An excerpt:

          Where, for starters, are the laws of quantum mechanics themselves supposed to have come from? Krauss is more or less upfront, as it turns out, about not having a clue about that. He acknowledges (albeit in a parenthesis, and just a few pages before the end of the book) that every­thing he has been talking about simply takes the basic principles of quantum mechanics for granted. “I have no idea if this notion can be usefully dispensed with,” he writes, “or at least I don’t know of any productive work in this regard.” And what if he did know of some productive work in that regard? What if he were in a position to announce, for instance, that the truth of the quantum-mechanical laws can be traced back to the fact that the world has some other, deeper property X? Wouldn’t we still be in a position to ask why X rather than Y? And is there a last such question? Is there some point at which the possibility of asking any further such questions somehow definitively comes to an end? How would that work? What would that be like?

          Never mind. Forget where the laws came from. Have a look instead at what they say. It happens that ever since the scientific revolution of the 17th century, what physics has given us in the way of candidates for the fundamental laws of nature have as a general rule simply taken it for granted that there is, at the bottom of everything, some basic, elementary, eternally persisting, concrete, physical stuff. Newton, for example, took that elementary stuff to consist of material particles. And physicists at the end of the 19th century took that elementary stuff to consist of both material particles and electro­magnetic fields. And so on. And what the fundamental laws of nature are about, and all the fundamental laws of nature are about, and all there is for the fundamental laws of nature to be about, insofar as physics has ever been able to imagine, is how that elementary stuff is arranged. The fundamental laws of nature generally take the form of rules concerning which arrangements of that stuff are physically possible and which aren’t, or rules connecting the arrangements of that elementary stuff at later times to its arrangement at earlier times, or something like that. But the laws have no bearing whatsoever on questions of where the elementary stuff came from, or of why the world should have consisted of the particular elementary stuff it does, as opposed to something else, or to nothing at all.

          Next, you write:

          Q. Since when did particles have characteristics of an intelligent being where a particle can observe things and have choices?
          A. Ever since they came into existence in our three dimensional universe. Please note that I am NOT claiming particles to be ‘beings’ of any sort, intelligent or otherwise. But they DO possess the ‘characteristics’ of intelligence in that, when ‘observed’, a quanta of mass (a ‘particle’) will be forced to ‘assume’ one of all of its possible ‘states’, an event characterized as being a ‘choice’ made ‘by the particle’ rather than an ‘effect’ imposed upon it by the observation. Also, the act of ‘observation’ in quantum mechanics is NOT intended (although in more complex systems may have a butterfly effect on the system as well) to mean actual viewing by a pair of human eyes. Rather, it connotes ‘measurement’ or ‘detection’ or any other method of attempting to ‘learn’ or ascertain ‘anything’ about the subject of the observation. The ACT of engaging in the attempt to gain information itself instantaneously causes the effect of ‘influencing the outcome of the observation’ in some way. In quantum mechanics, this is known as ‘decoherence’ (See, http://en.wikipedia.org/wiki/Quantum_decoherence ). In other disciplines, a similar effect is known as the Uncertainty Principal which states that you cannot know both the position and momentum of a particle at the same time, and, that the more certain you become about one, the less certain you become about the other (See, http://en.wikipedia.org/wiki/Uncertainty_principle ; http://en.wikipedia.org/wiki/Observer_effect_(physics) ).
          Although I have no citation for you to support my idea that this effect exists at the Plank scale between ‘stings’ of pure energy and a collection of two or more of them (something that is as yet ‘unproven’ to exist), it seems logical to presume such to be the case.

          You say that “the act of ‘observation’ is quantum mechanics is NOT intended to mean actual viewing by a pair of human eyes. The ACT of engaging in the attempt to gain information itself instantaneously causes the effect of ‘influencing the outcome of the observation’ in some way.”

          So how do you counteract my point in God Is Real…Why Modern Physics Has Discredited Atheism?:

          The simplest explanation of why modern physics has done away with materialism/naturalism is this: Material things cannot have “a complete, absolute independent reality in themselves” because, as modern physics has demonstrated, the material world cannot exist independent from consciousness (mind). There is no reality independent of mind.

          Here is how University of California, Berkeley physicist Henry Stapp puts it in his book Mindful Universe:

          “…According to contemporary orthodox basic physical theory, but contrary to many claims made in the philosophy of mind, the physical domain is not causally closed. [italics are his] A causally open physical description of the mind-brain obviously cannot completely account for the mind-brain as a whole.”

          “In short, already the orthodox version of quantum mechanics, unlike classical mechanics, is not about a physical world detached from experiences; detached from minds.”

          Princeton University quantum physicist Freeman Dyson echoes Stapp’s above comments:

          “Atoms are weird stuff, behaving like active agents rather than inert substances. They make unpredictable choices between alternative possibilities according to the laws of quantum mechanics. It appears that mind, as manifested by the capacity to make choices, is to some extent inherent in every atom. The universe is also weird, with its laws of nature that make it hospitable to the growth of mind. I do not make any clear distinction between mind and God. God is what mind becomes when it passes beyond the scale of our comprehension.”

          David, doesn’t “the ACT of engaging in the attempt to gain information,” (as you put it) require a mind…an agent? What could ACT to gain information other than a conscious agent? Prior to the existence of humans, what conscious agent was there to ACT?

          Once again, since your comment was so huge, it is entirely possible that I have missed responding to what you consider to be a substantive portion of your argument. If I have done so, please restate the substantive portions that I have missed in a clear and concise manner.

        • Graceus says:

          David,

          I am pleased to have introduced you to something new. However, I gathered from your assertion from point 1 “There is no god or creator” that you were an atheist, so I did not infer that from “atheists start out on this site using Bulverism, you used Bulverism, therefore you must be an atheist.” (Also, why would you try to persuade people to imagine that the reality is that there is no god unless you did not believe that God is real? So, yes, you did suggest or imply that there is no God). Nor did I engage in question begging when I cited your comments as Bulverism; the logic at how you think I came to my conclusion is wrong. From this definition from the Urban Dictionary, ‘Bulverism’ is the “The act of assuming without discussion that a person is wrong and then distracting his or her attention from this (the only real issue) by busily explaining why he or she is wrong and how that person became so silly.“ Or you can look ‘Bulverism’ up in Wikipedia. As you can see, your comments in the first half of both posts easily fall under this logical fallacy.

          Next, allow me to clarify my statement “By your logic, we could say that since scientists have been wrong in the past, they will therefore always be wrong in the future with their scientific theories and experiments. That’s illogical.” To clarify, I meant by your logic, scientists will _always_ be wrong in the future _100%_ of the time since they were wrong in the past. I am going to address this red herring fallacy: “faith based ‘answers’ to all manner of inquiry have always been proven to be WRONG, 100% of the time.” That’s generalizing all religious beliefs. Obviously, I do not agree that Christian beliefs have been proven wrong, but if you wish to continue down this path with statements like “This ‘phenomenon’ of systematically replacing ‘faith-based-solutions’ with those born of better reasoning producing a superior result is patently obvious”, then you will be continuing with your red herring fallacies that you started in your first post which takes up half your post, and this distracts from the actual topic. Recall that the topic of the debate is the origin of the universe, not psychoanalyzing why religious people believe what they believe or your implication that science has in the past proven religious beliefs wrong.

          Regarding time-all right, why don’t you clarify what the purpose of mentioning time is? Is this supposed to be evidence for something? Evidence for what? Time does not bring anything into existence, so please articulate how time caused the universe to come into existence. If time itself did not cause the universe to come into existence, I do not see the relevance in mentioning time.

          “I don’t want to get into any more detail than to just say I’m sorry if your faith insists on ‘keeping you in the dark’ so to speak, by clinging onto notions that are simply no longer valid”. What invalid notions? What are you presuming here? Or are you actually basing this on anything that I have actually said?

          “My answer is predominately ‘yes’. It did not arise ‘because’ of the quantum foam, but my idea is that it arose ‘from’ the quantum foam, as a consequence of the application of the laws of probability.”

          I have just read Scott’s reply to you, and he answers your belief quite well when he says that “Probability and time can never accomplish anything without 1) a causal mechanism and 2) an underlying structure, or order.” Time and probability are not causal agents.

          “Please, please, please take some time to learn about the quantum properties of matter…” I do not dispute that quantum processes exist. What is disputed are the interpretations and the conclusions drawn from those interpretations. But before I discuss quantum mechanics, allow me to address this comment “However, and contrary to his muse, physicists have more recently used the properties of quantum entanglement to transport photons over 88 miles, instantaneously, from one physical location to another”. Your insertion of this statement rests on hope (or faith) that one day information will be able to pass through black holes into another universe. As you correctly stated, Einstein may have the last laugh since there is a black hole information paradox, and each ‘solution’ to this has its advantages and disadvantages (http://en.wikipedia.org/wiki/Black_hole_information_paradox). By positing that statement, you have revealed to me that people will hold onto their theories not based upon evidence, but based on faith, even if it scientism (or faith in science)-that one day, science will be able to give us all the answers, and science may one day fill in all the gaps. This is called “Science/Nature of the Gaps”.

          “Q. Where did the ‘laws of nature’ come from?
          A. Other than those that _existed _prior_ to the singularity, they were all defined within the inflationary period that followed”. I am unaware of any laws of nature existing prior to the initial singularity (Big Bang). Where is the evidence that laws of nature existed prior to the Big Bang? I’m afraid that’s only an assumption that scientists believe in. I would like to see the evidence that justifies this assertion.

          Although there is no evidence that the laws of nature existed prior to the Big Bang, let’s just theorize that they did. Let’s also grant that quantum wave functions existed before the Big Bang, even though there is no evidence for that either. We know that there are 3 interpretations for QM: Copenhagen, Neorealist, and the Many Worlds Interpretations. *The Copenhagen interpretation is the standard or orthodox view. I am sure that you have heard of the popular theory that quantum wave functions collapse upon measurement since in your comments you attribute particles as being able to observe things. Dr. Roger Penrose, a colleague of Stephen Hawkings, I believe is credited with the “Consciousness causes collapse” theory. Basically, quantum wave functions collapse upon measurement; measurements are taken by observation, and observation is a function of the consciousness or the mind. So, if we take the Consciousness Causes Collapse theory and applied it to the beginning of the universe, what could have caused the first quantum wave function to collapse, which in turn started the chain of events for the origin of our universe when there was no living conscious human being at that time to take a measurement or observe a quantum wave function? God, whom Christians know as an intelligent, personal, omnipotent, creator and unembodied mind, would be the only being that could cause the first quantum wave function to collapse. Using abductive reasoning, we can infer that it was God’s consciousness. With regards to the Heisenberg uncertainty principle, that has to do with the RESULTS of observation of a quantum state; it is not causal!

          According to William R. Wharton, Professor of Physics at Wheaton College, who researched nuclear physics for eighteen years, quantum events, are not uncaused; quantum wave functions are not self collapsing. He believes that excluding causal chains would undermine quantum theory itself or make it difficult to correctly interpret. I quote his work here:

          “Because stationary states lack causal chains, the particle is _waiting_ for a causal chain, with a first _cause_ from outside, to give its non-conserved (accidental) properties a reality. All stationary state wave functions have a part which extends outside of the potential well, where the potential energy is greater than the total energy. The existence of the particle in this spatial region can only be made a reality if some outside probe, i.e. a scanning tunneling microscope, detects the position of the particle by imparting to it sufficient energy to actually exist where it is located. This is a first cause in a new causal chain; bringing reality to what previously was only a potentiality. To call this process tunneling through the barrier is a misnomer because the particle didn’t previously have a position inside the barrier. Similarly no bound particles in a stationary state can have momentum; otherwise they would be accelerating and giving off electromagnetic radiation, in contradiction to their stationary status. This rule that accelerating charge must emit or absorb radiation is no less sacred than is energy conservation. We must find an interpretation which holds both of these rules to be true. Whereas the absence of causality leaves the position and momentum of the electron undefined, its electric charge is usually causally related to the outside world. Therefore the spatial charge distribution must be a reality, distributed as a cloud given by the square of the QM wavefunction” (http://www.google.com/url?sa=t&rct=j&q=william+wharton%E2%80%99s+paper+causation+with+quantum+mechanics&source=web&cd=8&sqi=2&ved=0CF0QFjAH&url=http%3A%2F%2Fciteseerx.ist.psu.edu%2Fviewdoc%2Fdownload%3Fdoi%3D10.1.1.133.2208%26rep%3Drep1%26type%3Dpdf&ei=WUdjT8XWDOj50gGV_-mnCA&usg=AFQjCNF2eVH4RuPPMkcii8nzUlvIc_dYVA&cad=rja).

          There are criticisms to the “Consciousness causes collapse” theory. Here are two criticisms found in Wikipedia: “To many scientists this interpretation fails a priori to compete with other interpretations of quantum mechanics because ‘consciousness causes collapse’ relies upon a dualistic philosophy of mind (in particular, a radical interactionism), which is inconsistent with the materialist monism presupposed by many physicists.[16]” and ““It has been argued that [consciousness causes collapse] does not allow sensible discussion of Big Bang cosmology or biological evolution, at least on the assumption of an atheistic universe.”

          Notice the reasons for the problems is because it is “inconsistent with the MATERIALIST monism” and “does not allow for sensible discussion…on assumption of an ATHEISTIC universe.” So the problems listed do not have to do with science, but philosophical or metaphysical reasons.

          “Although I have no citation for you to support my idea that this effect exists at the Plank scale between ‘stings’ of pure energy and a collection of two or more of them (something that is as yet ‘unproven’ to exist), it seems logical to presume such to be the case. It has been shown that even the _slightest_ interaction_ between any ‘stuff’ massive enough to produce an output capable of being captured as ‘data’, even as ‘small’ as a pulse of light from a laser, WILL, more often than not, _cause_ an electron or even things as small as a single photon to be ‘forced’ to assume a particular state to the exclusion of all other possible states (make a choice”.

          You are forgetting that the ‘slightest interaction’ must be caused by something, and you then go on to say that the slightest interaction ‘caused’ an electron to assume a particular state. What ’caused’ the “slightest interaction”?

          “I can see no reason why the effect does not obey some scalar function”.

          And this is why Scott had said that it doesn’t matter which scientific theory you put forth. Because even if quantum processes were able to bring the universe into its own upon the laws of nature, all these would still have to answer where the quantum wave functions came from (which science has not answered; the skeptic’s answer that the laws of nature existed prior to the Big Bang is not based upon evidence, but upon faith) and where the laws of nature came from (which is also an assumption based upon faith). So, when you write about “energy, in its purest form, that obeys only the second law of thermodynamics as well as the laws of quantum mechanics and their attendant probabilities”, there is no evidence that energy, the second law of thermodynamics, and the laws of QM existed prior to the Big Bang, and belief in such is based upon faith. Naturalism has found to be lacking as the answer to the origin of the universe.

    • Tall Tom says:

      David,

      You wrote, “So, how about it? What if, just if, my little mental masturbation is correct? I understand that I have brutally bastardized a plethora of learned men’s life work, I can’t help it, I just ain’t smart enough to do the math. However, the principles I am describing are FACTS. They are not mere theoretical scribal on paper. They have, for the most part, been objectively verified in experiments that have yielded consistent results to the level that the scientific community was willing to publish many of the ideas AS FACTS and the others as ‘respected theories’ at the very least. Even if my observations are incorrect, inaccurate, and only ‘on the right track’, flat out wrong in may of the details, they nonetheless proceed in an objective and logical form with respect and due regard for the laws of nature as we best understand them today. My thought process sure seems to lead to conclusions that make far, far, far more ‘sense’ than that a guy with a beard did it all in 144 hours; that ‘He’ made man from two hunks of clay, and…

      First point. If you are “not smart enough to do the math” then you are not smart enough to understand the Science. I am sorry. Mathematics is the LANGUAGE OF PHYSICS and of the PHYSICAL SCIENCES.

      Second point. The principles which you are describing ARE THEORIES. Theories can be validated or invalidated by and through observation. There are no “facts” except that of the Empirical Evidence, the data, which can be used to validate or invalidate a theory. You really need a better understanding of the Scientific Method. What you write of as “theories” are what are commonly known as SPECULATIONS. Your misuse of terminology demonstrates deficiency and is suggestive that you are overwhelmed by the information you have read.

      Third point. Nobody in this dissertation has claimed that “a guy with a beard did it all in 144 hours; that ‘He’ made man from two hunks of clay.” That is what is called a Strawman Argument in debate.

      To attack an argument, effectively, one invalidates any of the premises of the argument. Then the conclusion of the argument is invalid.

      The argument presented did not have a God creating an Entire Universe in 144 hours, or , six days, as one of the premises.

      You need to look up and study the TRIVIUM. You also need to learn the rules of Logical Debate. Your lack of skills in this area is, once again, demonstrative of being overwhelmed by the topic at hand. Research the 42 Logical Fallacies as you have committed quite a few of them. Logical Fallacies are used by the IRRATIONAL…which leads me to the next point…

      You wrote, “If you have read this work and, if you have really tried to suspend your ‘belief’ long enough to entertain my effort as an alternative explanation for the nature and origin of the creation of our universe, at least THEORETICALLY capable of replacing your ‘faith based’ responses but just ‘can’t’ or continue to ‘refuse’ to do it, I suggest you get some help.”

      Well…I suggest that perhaps you need look in the proverbial mirror as your ad hominem attack demonstrates concession.

      I really do not care if you choose to bet the extremely long odds that there is not a God. Seriously I do not care whatsoever about your choice. What I do care about is that you wasted these people’s time while hypocritically claiming that their time would be better spent doing something else…

      There is a one in 10 raised to the 123rd power of a chance that you may be correct, that the Physical Universe is a product of a chance occurrence.

      But that is very slim odds. In fact that fraction is so close to ZERO that for all practical purposes it can be considered as ZERO.

      A probability of ZERO is known as an IMPOSSIBILITY.

      Of course that means that there is a 10 raised to the 123rd Power chance, minus one, in a 10 raised to the 123rd Power chance that the Universe was a Designed Manifestation. That number is so close to one that it might as well be one.

      A probability of One is known as a CERTAINTY.

      Do you even know who Roger Penrose is? He was Stephen Hawking’s contemporary on Black Holes.

    • John says:

      Hello David,
      I’m a 13-year-old in middle school. My entire life I have been a Christian, but only recently have I begun to consider myself devout. If you don’t take my opinion seriously because of my age, then so be it. I have a question for you. I came to this website to validate my belief in God. Why did you come to this website? Did you stumble on it from somewhere else in the internet? Did a friend send it to you in a link? Whatever the reason you made a voluntary action to open and read this article. In my mind, this is a calling from God. Many atheists that I have personally met defend their beliefs rigorously. No matter what kind of a conversation we have, they somehow manage to move the conversation into religion. This is one of many proofs of God. The reason you’re so bothered by other’s faith is not because they actually annoy you. You are being pestered by God. Somewhere in your subconscious, you made the decision to read this article. Something made you do this. You wrote that comment to reassure yourself of your beliefs (or lack thereof). I think that you thought many would have nothing to say about your claims. When we were all dumbfounded you would laugh and proclaim yourself winner of the argument. Now I will describe what your post actually did do.

      Starting off with claiming that religion causes violence. I do not deny that religious disputes have started conflicts. We see this clearly in wars such as the crusades. Instead, I would like to point you to the Bible. According to scripture, we are living in Satans world. God allows Satan to rule so that we can determine that God is a better fit for our needs. Obviously, Satan has done a horrible job. Look around at all the suffering if you want proof.

      Now when you describe the Quantum Foam, particles fading in and out in a matter of Planck time (This is the smallest unit of time for those that don’t know), don’t you think this sounds a little complicated. And by a little I mean VERY COMPLICATED. Tell me how this, along with the Anthropic Constants mentioned above, the perfect balance of dark energy, and many other Goldilocks-like effects around the universe.

      Thank you for reading. Again, if you don’t take me seriously, that’s your problem. I just want to give you something to chew on.

    • Patti Snee says:

      Hi David – I guess your hypothesis is about as reasonable as someone coming across this site on the web with all of these posts – including yours and taking the position that the alphabetical symbols on the screen somehow “chose” to arrange themselves according to certain rules and principles, in a very specific order, which then resulted in them actually being able to convey complex thought, ideas and information to something else??? That this happened independent of any intelligent being behind this process. So ok, maybe you’re right.

  4. […] OK…I Want Numbers. What is the probability that the universe is the result of chance?, I point out how the Oxford University mathematical physicist Roger Penrose calculated that […]

  5. Cliff says:

    In the last paragraph we seem to have moved from a scientific calculation to betting on a horse race – chance.

    Remember an argument is to decide who is right. A discussion is to decide what is right.

  6. Jim says:

    I have always wondered why atheists work so hard to convince others that God doesn’t exist. What is the “payoff”? Why the crusade?

    • Scott Youngren says:

      Jim,

      I used to wonder about the same thing. However, the more I read, and the more I listen to atheists, the more I am convinced that the answer to your question is best summed up in two words: MORAL ESCAPISM.

      In a nutshell, atheists find the idea that they have to answer to a higher power for their actions to be abhorrent and repugnant. Since humanity lives in a state of rebellion from God, we humans naturally prefer to be the kings of our own castles…answerable to no one. We prefer to do as we wish without any consequences and create our own morals.

      I wrote an entire essay about this topic titled If the Evidence for God Is So Strong, Why Are So Many People Unconvinced, which I recommend.

      Scott

      • Anon says:

        So the reason that I eat babies, dance naked under full moons and screw anything with a pulse is because I dont want to answer to anyone.

        I assume that this means that if I could conclusively prove that God didn’t exist then you to would act like this to because all morality comes from God, and without God, thats how you would behave

        You would not act like that, and nether do I. Why? Because morality does not work in the way taht you sugesst.

        • Scott Youngren says:

          Anon,

          The problem is that there can be no such thing as morality if we assume that atheism is true. For there to be an objective grounding for morality, there must be a source for objective morals. The natural world is has no morality. There is no such thing as a good or bad rock or a good or bad fish, etc.

          If human beings are the only source for morality, then there can only be subjective morality. Only if morals have an objective source above and beyond human opinion can they be said to be objectively true.

          If morals are only subjective than a statement such as the following has no objective truth: “It is morally wrong to commit genocide.”

    • Tyrhonius says:

      The reason for the “crusade” is to put an end to all the crusades have been undertaken in the name of God.

      • Scott Youngren says:

        Tyrhonius,

        If you would listen to the cries of history, you would quickly realize that it is when humans deny God that the real bloodbaths are started.

        The Communists were officially atheist and the Nazis, although not officially atheist, used atheist philosophy to justify their massive killing spree. Estimates of the number of people who died under atheist communism run as high as 110 million. This is an historically unprecedented number.

        I recommend my essay titled Doesn’t Religion Cause Killing?: https://godevidence.com/2011/02/doesnt-religion-cause-killing/

        …as well as From Darwin to Hitler by by professor of modern European history Richard Weikart.

        As Weikart points out, the racist Nazi rationalization for killing comes straight from Darwin. In The Descent of Man, Darwin writes:

        “With savages, the weak in body or mind are soon eliminated; and those that survive commonly exhibit a vigorous state of health. We civilized men, on the other hand, do our utmost to check the process of elimination; we build asylums for the imbecile, the maimed, the sick;….Thus the weak members of civilized societies propagate their kind. No one who has attended to the breeding of domestic animals will doubt that this must be highly injurious to the race of man.”

        At another point in The Descent of Man, Darwin writes:

        “The civilized races of man will almost certainly exterminate, and replace, the savage races throughout the world.”

        And exterminating everyone perceived to be “savage” or unworthy of passing on their genes is exactly what the Nazis tried to do. It is not, then, difficult to see why Weikart was justified in saying that:

        “Darwinism by itself did not produce the Holocaust, but without Darwinism…neither Hitler nor his Nazi followers would have had the necessary scientific underpinnings to convince themselves and their collaborators that one of the world’s greatest atrocities was really morally praiseworthy.”

        The following article also provides a great synopsis of how Darwinist philosophy led to the holocaust.

        http://www.trueorigin.org/holocaust.php

        Scott

      • Reasonable says:

        That view elimates the possibility of hope!

  7. Robin says:

    I look at it this way. Let’s say God doesn’t exist and that we just die. Then I would loose absolutely nothing by believing in what I believe. Let’s say that by me not believing in God and if the Bible is true, then, I would go to Hell and burn forever with demons torturing me and strung up in a cell begging God to forgive me. God would at that point be in tears and say, I’m sorry but their is nothing I can do to help you. While on earth, you had the choice weather to believe or not. This is the consequences of your actions. I came to life in the form of a human being, provided miracle after miracle, I was accused of Healing people on the wrong day. I had the choice to either stay here and get beaten to death so bad that I was unrecognizable or say forget it and leave. I sweated blood because as I was here on earth in the flesh and I felt the pain of the flesh but I did it on purpose because I Loved you all. I chose to do all this and knew ahead of time what I would have to go threw to save my people. Even tho my flesh died, I am still alive and well. I am still very sad that millions of people are still choosing not to believe. I cry everyday because of it. If your mother or father went through the same thing I did, how would you react. I Love you all and my intentions from the beginning of the earth was to create a beautiful, happy, loving, place here so life would be good.
    I don’t know about you, but I have experienced the power of God in my life through Miracle’s and have felt him spiritually and unless you are willing to experience what I have, You would have to believe. Or ask him to prove that he is real. But don’t misunderstand what you will experience with (science) It’s real so just admit it. Have a blessed day…

    • Tony Fordyce says:

      Oh, dear! Another person who wants to follow Pascal’s Wager. The main problem with that argument is that it means you have to believe in EVERY other religion too, in order to cover your bet and make sure that you are a believer in whatever (if any) religion turns out to be true (because it might not be christianity!). So are you a believer in Islam, Buddhism, Shinto, the ancient Greek and Inca gods, etc etc etc? Somehow I doubt it.

      • Scott Youngren says:

        Tony,

        You are taking this post out of the larger context in which it was written. In other words, this post was not meant to be a stand-alone argument for the Christian God. Rather, it was intended to rule out non-theistic religious belief systems such as atheism and agnosticism.

        This specific essay argues for a monotheistic God in general, but there are many essays at this site which argue for the God of the Bible in particular.

        You are creating what is known as a straw-man argument in philosophical terms by trying to suggest that I wrote this essay as a stand-alone argument for the Christian God. Simply put, you build a false characterization of my argument (a straw-man), and then proceed to attack that false characterization as if it were my actual argument…which it is not.

        Why don’t you read some of my essays which were intended to argue specifically for Christianity, and respond to them? Below are just 3 of many such essays:

        The Ancient Fable Behind Disbelief in Christ’s Resurrection.

        Do Christ’s Divinity and Resurrection Defy Common Sense?

        Is There A God? (What is the chance that our world is the result of chance?)

        Further, Buddhism is a non-theistic belief system. It does not comment on the existence or non-existence of God. Regarding polytheistic belief systems such as Shinto and the ancient Greek and Inca gods, Nancy Pearcey writes in her book Finding Truth:

        “…in polytheism, the personal gods and goddesses are not the ultimate reality because they derive from some pre-existing primordial substance— and that is what actually functions as the divine (even if the term is not used).

        For example, in ancient Greek mythology where did the gods come from? The essential story line is that the universe began as a divine primeval substance called chaos— an undefined, unbounded nothingness. Out of this initial state arose the first gods. The earth goddess (Gaia) mated with the sky god (Uranus) to produce the Titans, and from the mating of two Titans came the gods of Mount Olympus— Zeus, Apollo, Athena, Poseidon, and all the rest.”

        Under polytheistic belief systems, the universe produces the gods and not vice-versa. Therefore, the facts cited in this essay cannot be said to support polytheism (Greek gods, Shinto, etc).

        Scott

      • Tony says:

        No because the terms in the wager are simply God or no God. As such, someone who at least believes in some God would still be better off than a person who refuses, rejects and outright mocks God. So even based on your claim, it is better to be a theist than atheist in the end regardless.

        • Terry Collmann says:

          But that’s not what religions tell us, is it? They’re all very specific about how their god must be worshipped. They insist that a very specific god myst be chosen, and if you choosevthe wrong verdion, it’s off to hell

          • Scott Youngren says:

            Terry,

            The Christian Bible does not teach that people are sent to hell because they chose the wrong God. In fact, the Bible does not comment on what happens to people of other faiths.

            Rather, the Bible teaches that people wind up in hell as a just punishment for their sin. We are all sinners, if you think about it: Have you ever told a lie? I’ll bet you have, and this makes you a liar (and so am I). Have you ever wanted to kill someone? I’ll bet you have, and this means that you have murdered someone in your heart (and so have I).

            If a judge were to let a criminal such as a murderer or rapist off the hook without any punishment, would he be a just judge? No, he would be an unjust judge. God must punish sin in order to be a just God.

            But God is willing to pay the price for your sin himself, so that you can avoid your just punishment. He sacrificed himself on the cross so that you do not have to pay the price for your sin (hell). But you must accept this free gift by repenting of your sin and asking God to save you. You must put your faith in the sacrifice God made on the cross when he took on human form in Jesus Christ.

            In order to demonstrate that the concept of God sacrificing himself for our sins is not just a Christian concept, I wrote a post titled Which God is Real? In this post, I demonstrate the the Judeo-Christian concept of God is actually a concept that transcends culture and historical timeframe. I cite the Hindu scriptures known as the Vedas, which date to at least 1200 B.C., and the Upanishads, which date to around 500-400 B.C.:

            “The Supreme Creator took a perfect human body (Nishkalanka Purusha) and offered it up as a self-sacrifice (Brihad Aranyak Upanishad 1:2:8).”

            “If you want to be delivered from the sin, which you commit through eyes, mouth, ears and mind, bloodshed is necessary. Without shedding the blood, there is no remission for sin. That must be the blood of the Holy one. God is our creator. He is our King. When we were perishing, He came to save us by offering even his own body on our behalf.” (Tandya Mahabrahmana 4.15).

            “The sacrificial victim is to be crowned with a crown made of thorny vines.” (Rig-Veda 10:90:7, Brihadaranyakaopanishad 3:9:28).

            “His hands and legs are to be bound to a yoopa [a wooden pole] causing blood shed.” (Brihadaranyakaopanishad 3.9.28; Aitareya Brahmana 2:6).

            “The redemption is through shedding of blood only and that blood has to be through the sacrifice of God himself.” (Taittiriya Aranyaka, verse 3).

            “This [sacrifice] is the only way for the redemption and liberation of mankind. Those who meditate and attain this man, believe in heart and chant with the lips, get liberated in this world itself and there is no other way for salvation too.” (Yajur-Veda 31:18)

            “The Purusha was above sin, and only in knowing him does one attain immortality.” (Chandogyopanishad 1:6:6-7)

            “After giving Himself as the supreme sacrifice, this Purush resurrected himself.” (Brihadaranyakaopanishad 3.9.28.4-5; Kathopanishad 3: 15).

            “The purpose of this sacrifice is to provide the only way to Heaven and the only way to escape from Hell.” (Rig-Veda 9:113:7-11; 4:5:5; 7:104:3).

            “Before death he should be given a drink of somarasa [sour wine made of an herb called somalatha].” (Yajur-Veda 31).

            “None of His bones must be broken.” (Yajur-Veda 31:; Aitareya Brahmana 2:6)

            In addition to ancient Hindu culture, ancient Chinese culture (and many other cultures) also discuss God’s self-sacrifice for your sin. Please read the post I link to above.

            Scott

  8. Quora says:

    What are the mathematical proofs of Gods existence?

    Here is a list of 9 mathematical proofs for the existence (credibility, probability, and/or truthfulness of God): 1. Godel Part I. A recent discovery here: Holy Logic: Computer Scientists ‘Prove’ God Exists – SPIEGEL ONLINE You can read the paper her…

  9. Quora says:

    What should I do? My Muslim husband doesn’t let me eat pork, but I really want to.

    First off I’m going to as that we not continue this discussion here as it has no relevance to the question that was asked nor does it expand on or supplement the answer I gave. If you wish to have this argument/discussion I suggest you open your own q…

  10. […] Source: OK…I want numbers. What is the probability the universe is the result of chance? – God E… […]

  11. […] did a little digging online and found some research from John Lennox, Professor of Mathematics at Oxford […]

  12. […] did a little digging online and found some research from John Lennox, Professor of Mathematics at Oxford […]

  13. James says:

    I was wondering if I could save some of your articles such as this one to my hard drive?

  14. […] Here’s the link to the post from which I got my information, which goes into more depth, but the chances of this Earth happening on it’s own are: […]

  15. Nurul Kabir says:

    Useful information.

  16. Chris says:

    I thank you all for your logical approach using sound reasoning and the scientific method to show that the only rational explanation for our existence and that of the universe is that it was created by God. But you then let yourself down by illogically stating that God came down in human form,and that he tortures sinners in hell fire.
    However, I have read the Bible through and through. And have not read one verse that specifically states these things.
    But I have read many verses that state Jesus is God’s son,a being created by God,and that” the wages sin pays is death”. I am glad to provide these verses from any Bible translation. Please apply your logical fact based scientific argument to the scriptures and allow them to speak for themselves

    • Allan says:

      Ok… give me one verse that says Jesus is a created being. Do you understand what the reference/title Son of Man means? You need to look at the verses in an Ancient Near Eastern person’s perspective.

      We all die…why would the bible state a fact we all know already? Good people and bad people die.

      • Chris says:

        Collisions 1 vs 15 &16
        “He is the image of the invisible God. The firstborn of all Creation.Because by means of him all other things have been created I the heavens and on the earth. Other things have been created through him and for him”.
        Revelation 3 vs 14 ” These are the things that the amen says,the faithful and true witness, the beginning of creation by God”
        Proverbs 8 vs 22″ God produced me as the beginning of his way,the earliest of his achievements ”
        Vs 30 ” then I came to be beside him as a master worker”.
        Taken on face value,Reading it ,it shows quite clearly that God created his sonJesus Christ. Of course ,if people want to put their own spin on it without any clear scriptural support,I’m sure they will.
        But it’s a very dangerous thing to tamper with God’s word to try and bend it to fit our personal views instead of allowing it to speak for itself.

      • Chris says:

        Collisions 1 vs 15 &16
        “He is the image of the invisible God. The firstborn of all Creation.Because by means of him all other things have been created I the heavens and on the earth. Other things have been created through him and for him”.
        Revelation 3 vs 14 ” These are the things that the amen says,the faithful and true witness, the beginning of creation by God”
        Proverbs 8 vs 22″ God produced me as the beginning of his way,the earliest of his achievements ”
        Vs 30 ” then I came to be beside him as a master worker”.
        Taken on face value,Reading it ,it shows quite clearly that God created his sonJesus Christ. Of course ,if people want to put their own spin on it without any clear scriptural support,I’m sure they will.
        But it’s a very dangerous thing to tamper with God’s word to try and bend it to fit our personal views .

    • I highly recommend that you read my post titled Which God is Real? In this post, I demonstrate that references to God’s self-sacrifice on the cross is mentioned not only in the Bible, but in many other cultures. Please read the entire post, but here are some excerpts:

      Regarding Jesus, one crucial point is that Jesus’ life and purpose (to serve as God’s self-sacrifice in human form as a remission for human sin) was foretold before his birth. The 53rd chapter of the book of Isaiah from the Hebrew Bible, written 700 years before Jesus birth, is perhaps the most prominent of these prescient Hebrew scriptures:

      Isaiah 53
      1 Who has believed our message
      and to whom has the arm of the LORD been revealed?
      2 He grew up before him like a tender shoot,
      and like a root out of dry ground.
      He had no beauty or majesty to attract us to him,
      nothing in his appearance that we should desire him.
      3 He was despised and rejected by mankind,
      a man of suffering, and familiar with pain.
      Like one from whom people hide their faces
      he was despised, and we held him in low esteem.
      4 Surely he took up our pain
      and bore our suffering,
      yet we considered him punished by God,
      stricken by him, and afflicted.
      5 But he was pierced for our transgressions,
      he was crushed for our iniquities;
      the punishment that brought us peace was on him,
      and by his wounds we are healed.
      6 We all, like sheep, have gone astray,
      each of us has turned to our own way;
      and the LORD has laid on him
      the iniquity of us all.
      7 He was oppressed and afflicted,
      yet he did not open his mouth;
      he was led like a lamb to the slaughter,
      and as a sheep before its shearers is silent,
      so he did not open his mouth.
      8 By oppression[a] and judgment he was taken away.
      Yet who of his generation protested?
      For he was cut off from the land of the living;
      for the transgression of my people he was punished.[b]
      9 He was assigned a grave with the wicked,
      and with the rich in his death,
      though he had done no violence,
      nor was any deceit in his mouth.
      10 Yet it was the LORD’s will to crush him and cause him to suffer,
      and though the LORD makes his life an offering for sin,
      he will see his offspring and prolong his days,
      and the will of the LORD will prosper in his hand.
      11 After he has suffered,
      he will see the light of life and be satisfied;
      by his knowledge my righteous servant will justify many,
      and he will bear their iniquities.
      12 Therefore I will give him a portion among the great,
      and he will divide the spoils with the strong,
      because he poured out his life unto death,
      and was numbered with the transgressors.
      For he bore the sin of many,
      and made intercession for the transgressors.

      Perhaps most Christians reading this will already be familiar with the above prophecies. But, much less known is the fact that Jesus’ arrival was foretold in the Hindu scriptures. That’s right, Hindu scriptures. Sound like a bold claim? Decide for yourself:

      Roy Abraham Varghese cites excerpts from the Hindu scriptures known as the Vedas and Upanishads, which predate (by hundreds of years), and very convincingly seem to prefigure, the life and mission of Jesus, in his book The Christ Connection: How the World Religions Prepared the Way for the Phenomenon of Jesus. The Vedas date to at least 1200 B.C. and the Upanishads to around 500-400 B.C. Below are a few of these passages:

      “The Supreme Creator took a perfect human body (Nishkalanka Purusha) and offered it up as a self-sacrifice (Brihad Aranyak Upanishad 1:2:8).”

      “If you want to be delivered from the sin, which you commit through eyes, mouth, ears and mind, bloodshed is necessary. Without shedding the blood, there is no remission for sin. That must be the blood of the Holy one. God is our creator. He is our King. When we were perishing, He came to save us by offering even his own body on our behalf.” (Tandya Mahabrahmana 4.15).

      “The redemption is through shedding of blood only and that blood has to be through the sacrifice of God himself.” (Taittiriya Aranyaka, verse 3).

      “This [sacrifice] is the only way for the redemption and liberation of mankind. Those who meditate and attain this man, believe in heart and chant with the lips, get liberated in this world itself and there is no other way for salvation too.” (Yajur-Veda 31:18)

      “The Purusha was above sin, and only in knowing him does one attain immortality.” (Chandogyopanishad 1:6:6-7)

      “After giving Himself as the supreme sacrifice, this Purush resurrected himself.” (Brihadaranyakaopanishad 3.9.28.4-5; Kathopanishad 3: 15).

      “The purpose of this sacrifice is to provide the only way to Heaven and the only way to escape from Hell.” (Rig-Veda 9:113:7-11; 4:5:5; 7:104:3).

      “His hands and legs are to be bound to a yoopa [a wooden pole] causing blood shed.” (Brihadaranyakaopanishad 3.9.28; Aitareya Brahmana 2:6).

      “The sacrificial victim is to be crowned with a crown made of thorny vines.” (Rig-Veda 10:90:7, Brihadaranyakaopanishad 3:9:28).

      “Before death he should be given a drink of somarasa [sour wine made of an herb called somalatha].” (Yajur-Veda 31).

      “None of His bones must be broken.” (Yajur-Veda 31:; Aitareya Brahmana 2:6)

      And contrary to misunderstandings in the west (that have resulted from cultural mistranslations), Hinduism (at least as presented in the Vedas and Upanishads) is a largely monotheistic, not polytheistic religion. Varghese explains that “the tenth book of the Rig-Veda says, ‘The One Being is contemplated by the sages in many forms: Ekam santam bahudha kalpayanti.’”

      In The Wonder of the World, Varghese cites the great Hindu scholar B.N.K Sharma who declares that, also contrary to misinterpretations, Hinduism* is theistic as opposed to monist (theism teaches that God and the universe are separate, monism teaches that they are one in the same). Perhaps most remarkably, it is a monotheistic religion with echoes of the Christian concept of the Holy Trinity. Sound like more bold claims? Hard to believe? Read the book.

      Further, the God of the Bible apparently revealed what he was doing with the sacrifice of his son Jesus to the ancient Chinese. Chinese Christian Chan Kei Thong writes in Finding God In Ancient China: How the Ancient Chinese Worshiped the God of the Bible:

      “The Bible recounts a dramatic astronomic phenomenon occurring at the cruxifiction:

      It was now about the sixth hour, and darkness fell over the whole land until the ninth hour, because the sun was obscured; and the veil of the temple was torn in two. And Jesus, crying out with a loud voice, said, ‘Father, into Your hands I commit My spirit.’ Having said this, He breathed His last. (Luke 23: 44-46)

      Note that this solar eclipse lasted for three hours — from the sixth to the ninth hour, which is roughly noon to 3pm in modern timekeeping–before Jesus breathed His last. At that moment, the veil of the Jewish temple was supernaturally torn in two: a dramatic symbolization that the barrier between God and man was removed once and for all.”

      “Once again, this event is corroborated in the Chinese historical documents, which record a highly significant solar eclipse occurring around the time indicated in the biblical account:

      In the day of Gui Hai, the last day of the month, there was a solar eclipse, [The emperor] avoided the Throne Room, suspended all military activities, and did not handle official business for five days. And he proclaimed, ‘My poor character has caused this calamity, that the sun and the moon were veiled, I am fearful and trembling. What can I say?… Anyone who presents a memorial is not allowed to mention the word ‘holy’”.

      “Another entry made a short time later, referring to the same eclipse, said:

      Summer, fourth month [of the year], on the day of Ren Wu, the imperial edict reads, ‘Yin and Yang have mistakenly switched, and the sun and the moon were eclipsed. The sins of all the people are now on one man. [The emperor] proclaims pardon to all under heaven.’

      This solar eclipse was recorded in the Record of the Latter Han Dynasty, Gui Han was the last day of the third month in the spring, during the 7th year of Han Emperor Guang Wu (reigned A.D. 25 – 57). That corresponds to A.D. 31, which means that this major eclipse happened 34 years after the astral events involving the magi! “[At the time of Christ’s birth]

      “Even more incredibly, a commentary in the Record of the Latter Han Dynasty, said simply,

      ‘Eclipse on the day of Gui Hai, Man from heaven died.’

      The man from heaven died! Could there be a more apt description or a more accurate understanding of the Cruxifiction?

      God’s love extends to all the nations of the world, and in some unique and unknown way, He gave special insight to Chinese astronomers to understand what He was doing. We are all sinners, even the emperor, but God laid all our sins on His only Son, Jesus Christ.”

      Thong then goes on to describe how the Record of the Han Dynasty records that a rainbow encircled the sun three days after the eclipse, which corresponds with the day of Christ’s resurrection.

      Perhaps these are contributing factors to the phenomenon of China already having more Christians than Communist Party members!

  17. David Platek says:

    God? Which one? How many have there been since 4000 b.c.? News flash: There is no such thing as a ‘god’. Grow up already. Start using your mind instead of living in the childlike world of ‘belief’. How many humans have been murdered in the name of a ‘god’ or because of what they ‘believed’? Compare that number to the number of atheists who have ‘killed’ in the name of science.

    For all you ‘believers’: Never call upon a doctor, pray to your god when you break your arm; When your house starts on fire, just drop to your knees, ‘He’ will surely take care of you, and; so on and so on and so on.

    Oh, and which version of the bible is the ‘right one’ these days? Can one of you clowns explain to me how there can be a debate about “the word of God”?

    You’re all pittiful. Wasting away your own intelligence on childish notions.

    • You commit a Fallacy of Equivocation when you speak of human concepts of God (gods) as if they were the same thing as the Being who is the ground of all existence (God). These are two entirely different things.

      Please note that concepts of God are not mutually exclusive. For example, I am a Christian, but I think that the Muslims, Jews, and Hindus get an awful lot RIGHT with their concepts of God. All of these faiths regard God as the creator of the universe, the ultimate judge of morality, and as a single, infinite being, etc..

      Yes, there are a multitude of different human concepts of God. However, there are also a multitude of different concepts of what happened on the day of the JFK assassination. Some people think Lee Harvey Oswald acted alone, whereas others think there was another shooter hidden near the grassy knoll. Some people think there was a conspiracy led by the CIA (Russians, mafia, Cubans, Lyndon Johnson, etc.), whereas others think that there was no conspiracy.

      But does this multitude of human concepts invalidate the underlying reality of the JFK assassination? To suggest as such would be a non-sequitur (Latin for “does not follow”). It is certainly true that some of the concepts of what happened that day in 1963 are more likely to be accurate than others. In order to determine which concept is most correct, one must employ one’s reason. Beneath these diverse concepts is an agreement of an underlying reality that JFK was murdered that day in Dallas.
      Regarding polytheistic belief systems such as the ancient Greek mythology which includes the god Zeus, Nancy Pearcey writes in her book Finding Truth:

      “…in polytheism, the personal gods and goddesses are not the ultimate reality because they derive from some pre-existing primordial substance— and that is what actually functions as the divine (even if the term is not used).

      For example, in ancient Greek mythology where did the gods come from? The essential story line is that the universe began as a divine primeval substance called chaos— an undefined, unbounded nothingness. Out of this initial state arose the first gods. The earth goddess (Gaia) mated with the sky god (Uranus) to produce the Titans, and from the mating of two Titans came the gods of Mount Olympus— Zeus, Apollo, Athena, Poseidon, and all the rest.”

      Under polytheistic belief systems, the universe produces the gods and not vice-versa. Therefore, comparing monotheistic concepts of God (the infinite being who is the ground of everything) with polytheistic gods (which are the product, rather than the cause of the universe) is uninformed and confused reasoning.

    • Chris says:

      Hi David. You make some valid points. It’s no wonder people like you are put off religion,God,Christianity.
      You say how can there be a debate about the word of God.
      Confusion reigns among professed believers,,because……
      They invalidate the plain truth in the Bible by their own religious traditions .Traditions which are rooted in pagan worship,,and infiltrated their way into true Christianity. Jesus himself even warned this. Matthew 15 vs 7.”
      You hypocrites,Isaiah aptly prophesied about you when he said” these people honour me with their lips but their hearts are far removed from me. It is in vain that they keep worshipping me for they teach commands of men as doctrine”.
      Mark 7 vs 13 ” Thus you make the word of God invalid by your traditions that you have handed down. ”
      So we end up with thousands of confusing religions. Hundreds of various offshoots of supposed Christianity.
      There is only one God,the creator of all things. One mediator between God and man..Jesus Christ. One true form of worship.
      The one which God’s own word clearly describes to us throughout its pages.
      David,,if you want the truth,,study carefully the Bible yourself. Cross reference verses. Make a note of those which are simple,straightforward and unambiguous and cannot be misinterpreted . Other verses that could be misinterpreted,,find every other verse or section of scripture that deals with that subject,cross reference until you have exhausted it.
      There is so much falsehood in Supposed Christianity today.
      Check any encyclopedia, and look up the origins of so called Christian holidays like Christmas and Easter. Check out roman emperor contstantine and how he combined roman traditions with Christianity to form wat became the holy roman empire.
      Most Christian denominations have paid clergy with special garb and titles.
      Where did that idea come from?
      Not from Jesus. He said you received freely,give freely.
      READ YOUR BIBLE.
      check various translations against each other to find the most consistent translation of the verses.
      The one true God will not forever tolerate these confusing mish mash of false Christianity.
      The Bible makes that very clear.
      If you want to ask any specific question at all, I will point to the relevant scriptures to answer you,,not my own viewpoint

  18. Greg says:

    Hi everyone. Love all the comments especially those Chinese records speaking of Christ. One thing I’ve learned in my study of our loving Father in Heaven, is that He loves ALL His children and because one has found God now or 50yrs ago or at sometime in the future doesn’t change His love for them. The reality of God doesn’t change with the whims of man’s belief or lack thereof. God’s reality is a foundational eternal truth. Evidence of His eternal love is He allows us to learn when we’re humble and ready. So He has a way of tenderizing us over time as we come to understand “that we can merit nothing without His help”.
    And on the thought of other countries knowing about Christ before His coming; I’ve believed that from a very early age. I have scripture that speaks of that.
    2Nephi29:8-12
    Wherefore murmur ye, because that ye shall receive more of my word? Know ye not that the testimony of two nations is a witness unto you that I am God, that I remember one nation like unto another? Wherefore, I speak the same words unto one nation like unto another. And when the two nations shall run together the testimony of the two nations shall run together also.
    9 And I do this that I may prove unto many that I am the same yesterday, today, and forever; and that I speak forth my words according to mine own pleasure. And because that I have spoken one word ye need not suppose that I cannot speak another; for my work is not yet finished; neither shall it be until the end of man, neither from that time henceforth and forever.
    10 Wherefore, because that ye have a Bible ye need not suppose that it contains all my words; neither need ye suppose that I have not caused more to be written.
    11 For I command all men, both in the east and in the west, and in the north, and in the south, and in the islands of the sea, that they shall write the words which I speak unto them; for out of the books which shall be written I will judge the world, every man according to their works, according to that which is written. 12 For behold, I shall speak unto the Jews and they shall write it; and I shall also speak unto the Nephites and they shall write it; and I shall also speak unto the other tribes of the house of Israel, which I have led away, and they shall write it; and I shall also speak unto all nations of the earth and they shall write it.
    As you can tell this isn’t someone speaking of Christ, this is the Savior speaking to us. There will come a future day when all truth will be revealed and it won’t be a battle of who’s right it will be a revelation of all truth. So when we are tempted to take up weapons(words) against those who may believe a little different than us we may want to follow the example of Gamaliel in Acts5:38-39.
    38 And now I say unto you, Refrain from these men, and let them alone: for if this counsel or this work be of men, it will come to nought:
    39 But if it be of God, ye cannot overthrow it; lest haply ye be found even to fight against God.
    Also God evidence falls into the same group as those who want a sign before they believe. One of the purposes of our earthly sojourn is to test our faith
    And he that seeketh signs shall see signs, but not unto salvation.
    8 Verily, I say unto you, there are those among you who seek signs, and there have been such even from the beginning;
    9 But, behold, faith cometh not by signs, but signs follow those that believe.
    10 Yea, signs come by faith, not by the will of men, nor as they please, but by the will of God.
    11 Yea, signs come by faith, unto mighty works, for without faith no man pleaseth God; and with whom God is angry he is not well pleased; wherefore, unto such he showeth no signs, only in wrath unto their condemnation.
    12 Wherefore, I, the Lord, am not pleased with those among you who have sought after signs and wonders for faith, and not for the good of men unto my glory.
    I just want to thank everyone for your thoughtful answers and pray that all honest seekers of truth will endure to the end when all truth will be revealed and we are unified in truth and faith.

    • Greg,

      Amen to that! If you are interested in other cultures having knowledge of the God of the Bible and Jesus Christ, you simply MUST read Eternity In Their Hearts by Don Richardson, a Christian missionary.

      I cite Richardson’s book in my essay titled Which God is Real?. An excerpt:

      This phenomenon of the one true God revealing himself to all peoples is further described by Christian missionary Don Richardson in his book Eternity In Their Hearts: Startling Evidence of Belief in the One True God in Hundreds of Cultures Throughout the World.

      Among the many examples that Richardson cites is that of the Karen people of Burma (now known as Myanmar). He cites Karen hymns as evidence to this fact:

      One such hymn extolled the eternity of Y’wa’s being [Y’wa being the Karen name for God]:

      “Y’wa is eternal, his life is long. One aeon–he dies not! Two aeons–he dies not! He is perfect in meritorious attributes. Aeons follow aeons–he dies not.’”

      The Karen story of man’s falling away from God contains stunning parallels to Genesis chapter 1:

      “Y’wa formed the world originally. He appointed food and drink. He appointed the “fruit of trial.” He gave detailed orders. Mu-kaw-lee [Satan] deceived two persons.”

      “When Y’wa made Tha-nai and Ee-u, he placed them in a garden…saying, ‘In the garden I have made for you seven different kinds of trees, bearing seven…kinds of fruit. Among the seven, one tree is not good to eat…If you eat, you will become old, you will sicken, you will die…Eat and drink with care. Once in seven days I will visit you…’”

      “After a time Mu-kaw-lee came to the man and woman and said, ‘Why are you here?’

      “Our father put us here,’ they replied.

      “What do you eat here?’ asked Mu-kaw-lee

      “Our Lord Y’wa has created food for us, food without limit.’

      “‘Show me your food,’ said Mu-kaw-lee.

      “…Our Father, the Lord Y’wa said to us, ‘Eat not the fruit of this tree. If you eat, you will die.’

      “…Then Mu-kaw-lee replied, ‘It is not so, O my children. The heart of your Father Y’wa is not with you. This is the richest and the sweetest…If you eat it, you will possess miraculous powers. You will be able to ascend to heaven… If you will eat the fruit as a trial, then you will know all…’”

      In the paragraphs that follow, the man, Tha-nai, refuses the enticement and walks away. The woman, Ee-u, lingers, succumbs to temptation, eats the fruit and then entices her husband, who also eats.

      Richardson goes on to describe how these Karen beliefs about Y’wa almost certainly “predate both Judaism and Christianity.” Later in the book, after detailing many other cultures with concepts of God strikingly similar to the Judeo/Christian God, Richardson laments:

      “How tragic then that Christians in general have been told almost nothing of this worldwide phenomenon of monotheistic presupposition underlying most of the world’s folk religions! Many theologians — and even some missionaries whose ministries have been tremendously facilitated by the phenomenon — have nervously pushed this mind-expanding evidence into the closet.”

      “Why? If you belong to a tradition which has been teaching Christians for centuries that the rest of the world sits in total darkness and knows zilch about God, it becomes a little embarrassing to have to say, ‘We have been wrong. In actual fact, more than 90 percent of this world’s folk religions acknowledge at least the existence of God. Some even anticipate His redeeming concern for mankind.’”

      “…No other message on Earth has an inside track already laid for it in the belief systems of thousands of very different human societies!”

  19. Logan says:

    Hello! I’m wondering how exactly did Penrose come up with the number 10^10^20. I came across a Quora answer which raises interesting questions about this number:

    “The number that’s been called the Penrose number is 10^10^123.

    It is the culmination of computations that appear in Roger Penrose’s 1989 book The Emperor’s New Mind in the chapter entitled “Cosmology and the arrow of time”, page 344.

    “Penrose argues that the number V of the volume of the phase space of possible universes is V=10^10^123. Further, he argues that among those universes, the volume W of those universes compatible with the second law of thermodynamics and with what we now observe is W=10^10^101. Therefore the probability that a universe is uniformly chosen at random among all the possible universes so that the second law of thermodynamics holds and appears like our universe is the quotient W/V=1/10^(10^123−10^101). That’s approximately 1/10^10^123 since 10^101 is infinitesimal with respect to 10^123.

    Let’s leave aside the computations that led to these numbers and consider the form of the argument.

    The argument starts with the volume of the space of all possible universes. Perhaps the sample space is incorrect. There may be some physical restriction on the possible universes. One single equation necessary for the formation of the universe could shrink this volume down to an infinitesimal fraction in the same way that the area of earth’s equator is infinitesimal compared to the area of the entire earth’s surface. Penrose uses this explanation himself when he proposes his “Weyl Curvature Hypothesis” as a constraint for the formation of the universe.

    Next, there’s the question of the validity of uniform probability. Is there any reason to believe that our universe was chosen from all possible universes uniformly? The reason for making a uniform assumption is from a lack of knowledge; we don’t know a better distribution to use. But cosmology is a young field, and there are new developments in the physics of the early universe.

    Finally, there’s the conclusion: the probability that we got the universe we did get is infinitesimal. But that’s always the way it works. The probability that any particular person wins the lottery is infinitesimal, but someone does anyway. If the universe is going to be chosen from all possible universes, any particular kind is extremely unlikely, but some kind will be chosen. In this case, only the winner survives to be an observer.

    Unfortunately, Penrose used some flowery language in his book: “This now tells us how precise the Creator’s aim must have been: namely to an accuracy of one part in 10^10^123.
    This is an extraordinary figure…” Because he used the word Creator, that passage has been taken by Creationists as evidence that the universe was created by God. If they had just read another paragraph, they could have seen how a Weyl Curvature Hypothesis would make unnecessary that conclusion.”

    Another confutation I came up with is: what’s the probability that pi is exactly 3.14159…? The question doesn’t make sense, as it is impossible to have a different ratio between the circumference and diameter of a circle. What if, like pi, the universal constants are what they are because of necessity, and not probability?

    • The argument starts with the volume of the space of all possible universes. Perhaps the sample space is incorrect.”

      But, here, you are building an argument for an unintelligent, materialist explanation for the origin of the universe based upon what we do not know, but may know some bright and shining day in the future. All explanations either for or against God must be based upon current knowledge, not hypothetical future knowledge. Arguments based on hypothetical future knowledge are faith based arguments, not fact based arguments (which is exactly what materialists frequently accuse theists of). And this is what the prominent philosopher of science Karl Popper was referring to when he coined the derisive term “promissory materialism.”

      Promissory materialism

      An excerpt from the above article:

      “Something similar is happening with materialism. Since the nineteenth century, its advocates have promised that science will explain everything in terms of physics and chemistry; science will show that there is no God and no purpose in the universe; it will reveal that God is a delusion inside human minds and hence in human brains; and it will prove that brains are nothing but complex machines.”

      “Materialists are sustained by the faith that science will redeem their promises, turning their beliefs into facts. Meanwhile, they live on credit. The philosopher of science Sir Karl Popper described this faith as ‘promissory materialism’ because it depends on promissory notes for discoveries not yet made. Despite all the achievements of science and technology, it is facing an unprecedented credit crunch.”

      Next, there’s the question of the validity of uniform probability. Is there any reason to believe that our universe was chosen from all possible universes uniformly? The reason for making a uniform assumption is from a lack of knowledge; we don’t know a better distribution to use. But cosmology is a young field, and there are new developments in the physics of the early universe.”

      This is another such attempt to build an argument for a materialistic (atheistic) explanation for the origin of the universe which relies on credit (a promissory note).

      Finally, there’s the conclusion: the probability that we got the universe we did get is infinitesimal. But that’s always the way it works. The probability that any particular person wins the lottery is infinitesimal, but someone does anyway. If the universe is going to be chosen from all possible universes, any particular kind is extremely unlikely, but some kind will be chosen. In this case, only the winner survives to be an observer.”

      I’m glad you used the example of the lottery. It is fallacious to argue that randomness has produced anything, ever. Period.

      As you correctly point out, he odds that a particular individual will win the lottery are incredibly small, but many people have won the lottery in the past, and many will win the lottery in the future.

      However, it would be absurd to suggest that anyone ever won the lottery purely as the result of chance or randomness. This is because randomness only works upon an underlying structure or order. In the lottery example, there must be the underlying structure of a lottery commission, and a distribution network for lottery tickets. And there must also be the underlying structure of a monetary system (Dollar, Euro, etc). Without this underlying structure, the odds of anyone winning the lottery, ever, are exactly ZERO.

      Like a lottery win, the origin of the universe requires an underlying structure. How did this structure originate? Why is reality structured so that a universe can occur? This is an ontological or meta-scientific question, not a strictly scientific one. And both theism and atheistic materialism are ontological or meta-scientfic stances. Ontology is the branch of philosophy which considers the nature of being, and meta is the Greek word for beyond or after.

      All that pointing out the role of probability does is kick the can down the road by sweeping under the rug the question of who or what is responsible for this underlying order or structure upon which randomness operates. An Acme brand universe generating machine?

      What if, like pi, the universal constants are what they are because of necessity, and not probability?

      Be careful not confuse necessary conditions with a causal explanation. To see part of the flaw in this reasoning, consider the following: Imagine that an insurance company sends an insurance investigator to find out why a warehouse burned down on a rainy night. After surveying the charred remains of the building, the investigator submits his report. The building burned down, he confidently explains, because of oxygen in the atmosphere.

      The insurance company promptly fires the investigator and hires someone else. What was his mistake? The investigator confused a necessary condition of fire with the cause of the specific fire under investigation. In this case, the oxygen was not the “difference that made a difference”—that is to say, the cause of the fire. Oxygen had been in the air around the warehouse all along. Its presence was not by itself sufficient to produce the fire and thus does not explain why the building suddenly burned down. The gas cans and matchbooks left near the scene of the charred building would have provided more relevant information, had the investigator not been so literally “clueless.”

      Oxygen is indeed a necessary condition of fire, but saying so did not provide a causal explanation of the particular fire in question. Similarly, the fine tuning of the physical constants is a necessary condition for life, but that does not provide a causal explanation of, or eliminate the need to explain, the fine tuning itself.

Leave a Reply

Your email address will not be published. Required fields are marked *